Exam 3 Adv Health Assessment

Ace your homework & exams now with Quizwiz!

True or False? The urethra carries urine from the bladder to the outside of the body, and extends through the length of the penis.

True

Which of the following tests is used to diagnose meningitis? A) CT scan B) Lumbar puncture C) Carotid angiography D) Ultrasound

B) Lumbar puncture

Which of the following is a sign or symptom of juvenile rheumatoid arthritis? A) Fractures B) Changes in pulses and sensation C) Bowel/bladder dysfunction D) Dyspnea

D) Dyspnea

What cultures and smears should be performed during the pelvic examination, and in which order?

(1) Culture samples for STDs (2) Cultures for infection (3) Pap smear

True or False? A patient's elbow should not be able to flex to 90 degrees.

False

True or False? Bubble baths and regular douching can prevent allergic vaginitis.

False

What are the six common back problems listed in the text?

- Compression fracture/back pain - Low back strain - Malignancy/tumor - Ankylosing spondylitis - Cauda equina syndrome - Herniated disc

Normal range for urinalysis specific gravity

1.003-1.025

Normal range for urinalysis sodium level

20 mEq/L

Normal range for urinalysis pH

4.5-8

A patient complains of unrelenting back pain. Resting does not help; neither do stretching and exercise. What is most likely? A) A nonmusculoskeletal issue B) A herniated disc C) Muscular rupture D) Osteoporosis

A) A nonmusculoskeletal issue

True or False? A patient who has begun menopause at 55 years has a higher risk of breast cancer.

True

A bluish hue noted in the cervix is most likely indicative of what? A) Early pregnancy B) Anemia C) Malignancy D) Hypoxia

A) Early pregnancy

True or False? Damage to the temporal lobe of the brain may leave a patient with impaired hearing as well as emotional/behavioral abnormalities.

True

On a history and physical examination of a 34-year-old patient, during the review of systems, he reports occasional episodes of headache. He describes the headache pain as a "band is around his head" and tightness in the neck and shoulders. The clinician should recognize these symptoms as: A. Tension headache B. Cluster headache C. Sinusitis D. Migraine headache

A. Tension headache Typical symptoms of tension headache include mild to moderate nonthrobbing pressure, or squeezing pain, that can occur anywhere in the head or neck. The pain often starts slowly as a dull and aching discomfort that progresses to holocranial pain and pressure. The pain can recur intermittently, lasting from minutes to hours, usually remitting with rest or removal of the stressful trigger. There is usually no associated nausea and vomiting. Although patients may report photophobia and phonophobia, it is less severe than those associated with migraines. Tension headaches are not aggravated by movement or activity. The neck muscles are often tight to palpation.

Your 26-year-old female patient reports a history of herpes simplex type II infection. On physical examination, you note red blisters over the vulva region. When questioned about sexual activity, she explains that her partner uses a condom whenever her infectious blisters are present. The clinician should recognize: A. The herpes virus can be transmitted during the patient's asymptomatic periods B. If the lesions are herpes simplex type I, then the virus is not contagious C. Only herpes simplex type II infects the genital region, and it is always contagious D. When the blisters rupture, viral particles are transmitted to a sexual partner

A. The herpes virus can be transmitted during the patient's asymptomatic periods Herpes I or II can infect the pelvic region and they cause the same syndrome. Approximately 10% to 15% of persons who carry the herpes virus are asymptomatic, but they can still shed the virus— thus, referred to as ―asymptomatic shedders.‖ This, among other things, contributes to the spread of the herpes virus.

Which is the major weight-bearing bone in the foot? A) Calcaneus B) Talus C) Malleoleus D) Navicular

B) Talus

Which of the following is generally palpable? A) The left kidney B) The right kidney C) The bladder D) The ureter

B) The right kidney

A 32-year-old female reports normal monthly menstrual periods but complains of having trouble with conception. It is important to ask about history of: A. Dysmenorrhea B. Sexually transmitted disease (STD) C. Menorrhagia D. Pap smears

B. Sexually transmitted disease (STD) Infertility may be caused by anovulation, decreased function of the corpus luteum, or blocked or scarred fallopian tubes. Any of these can occur despite the fact that the patient may be having menstrual cycles. A history of menstrual irregularities may suggest anovulation or thyroid disease, either of which can cause infertility. A history of STDs could lead to scarring of the fallopian tubes, which can cause infertility.

Which DTRs are tested in a reflex test?

Biceps, brachioradialis, triceps, patellar, achilles

What produces sperm? A) The penis B) The epididymes C) The testicles D) The prostate gland

C) The testicles

Which of the following often present with intermittent neurological deficit, vertigo, and weakness? A) Migraines B) Cluster headaches C) Transient ischemic attacks D) Tension headaches

C) Transient ischemic attacks

Palpating the masseter and temporal muscles tests which cranial nerve? A) II B) IV C) V D) VIII

C) V

What does epididymitis often present with? A. Pain and prepuce swelling B. Edema, dysuria, and pruritis C. Scrotal pain, edema, possible fever

C. Scrotal pain, edema, possible fever

How many bones make up the vertebral column? A) 37 B) 24 C) 12 D) 26

D) 26

What different medical problems can cause a breast lump?

Fibroadenoma, fibrocystic disease, fat necrosis, abscess, or cancer

Having a patient identify a physical object by touch with the eyes closed tests what? A) Graphesthesia B) Vibratory sense C) Extinction D) Stereognosis

D) Stereognosis

What are the three likely assessment findings for a patient with a dislocated shoulder?

- Any trauma to the shoulder with a mechanism of injury involving external rotation with abduction - Loss of the normal contour of the shoulder (X-rays will confirm) - Marked instability: when moving anteriorly, the shoulder moves too far forward and when moved posteriorly, it moves too far back

The internal os is contained within what? A) The uterus B) The vagina C) The ovaries D) The clitoris

A) The uterus

Which of the following bones is part of the wrist? A) Ulna B) Scaphoid C) Phalanges D) Metacarpals

A) Ulna

Which of the following disorders can cause urinary incontinence? A. Cystocele B. Overactive bladder C. Uterine prolapse D. All of the above

D. All of the above Cystocele and prolapse of the uterus can affect pelvic floor anatomy and cause urinary changes, including incontinence and changes to the quality of the urinary stream. Overactive bladder is a cause of incontinence due to atrophic changes of the pelvic floor muscles.

A bluish hue to a cervix is considered normal in which of the following? A) A prepubescent girl B) A pregnant woman C) Any woman of child-bearing age D) A geriatric patient

B) A pregnant woman

Which of the following is a sign of LMN lesion? A) Muscle spasticity B) Absent Babinski reflex C) No fasciculations D) Diminished strength, but no atrophy

B) Absent Babinski reflex

What is the purpose of the areola? A) An extended visual marker for a newborn or infant to latch onto B) Contains glands that secrete a lipid to lubricate the area during lactation C) No known physiological purpose D) Contains the mammary glands

B) Contains glands that secrete a lipid to lubricate the area during lactation

What would be an abnormal finding when inspecting the breasts? A) Supernumerary nipples B) Discharge without palpation in a nonpregnant patient C) Seborrheic keratosis D) Nevi and striae

B) Discharge without palpation in a nonpregnant patient

The Thomas test is used in assessing what? A) Shoulder range of motion B) Hip/pelvis range of motion C) Arm sensory loss D) Foot pain

B) Hip/pelvis range of motion

What would be a normal finding when inspecting the breasts? A) A unilateral predominant venous system B) Montgomery tubercles C) Dimpling D) Ecchymosis

B) Montgomery tubercles

Sarcoidosis is an autoimmune disease that presents with non-specific symptoms of fatigue, fever, and arthralgias. Which of the following signs should raise suspicion of this disorder? A. Facial rash across nose and cheeks B. Bilateral hilar lymphadenopathy on chest x-ray C. Specific tender points on the body D. Swelling of metacarpophalangeal joints

B. Bilateral hilar lymphadenopathy on chest x-ray Sarcoidosis is an inflammatory disorder in which patients develop granulomas and a wide range of symptoms, including arthritis. It is most commonly diagnosed in persons between ages 20 and 40. Arthralgias occur in approximately 3% of patients with sarcoidosis, and the most commonly affected joints include the ankles, feet, and hands. The patient may complain of constitutional symptoms, including fatigue, fever, and altered appetite. Respiratory symptoms, including cough, wheezing, and shortness of breath, are primary symptoms. On chest x-ray, hilar lymphadenopathy and pulmonary granulomas are key diagnostic signs.

On a physical examination for employment, a 45-year-old male shows no significant findings and takes no medications. Past medical history and surgery are unremarkable. On urinalysis, hematuria is present. The urinalysis is repeated on another day and still reveals microscopic hematuria. It is important to recognize that painless hematuria can be diagnostic of: A. Urinary tract infection B. Bladder cancer C. Nephrolithiasis D. Pyelonephritis

B. Bladder cancer Approximately 80% to 90% of patients with bladder cancer present with painless gross hematuria. All patients with this classic presentation should be considered to have bladder cancer until proof to the contrary. Patients should be referred to a urologist. They usually undergo cystoscopy to rule out bladder tumor. The majority of patients with bladder cancer present with no urinary tract symptoms or palpable masses.

Which of the following is true of UTIs in pediatric patients? A) They can occur before the age of 2 years. B) Children younger than age 6 years do not get UTIs. C) In adolescents, increased UTIs suggest sexual activity. D) They may have decreased urinary output.

C) In adolescents, increased UTIs suggest sexual activity.

Your 66-year-old male patient has recently started treatment for metabolic syndrome and is currently taking the following medications: an ACE inhibitor and beta blocker for treatment of hypertension. He is also taking a statin medication, simvastatin for hyperlipidemia, and a biguanide, metformin, for type 2 diabetes. The patient complains of myalgias of the legs bilaterally and blood work shows elevated serum creatine kinase. Which of the medications can cause such a side effect? A. Beta blocker B. ACE inhibitor C. Statin medication D. Metformin

C. Statin medication Myalgias are a side effect of lipid-lowering medications called "statins." If the myalgia is related to rhabdomyolysis, the urine is often reddish-brown. When drug-induced myalgia is present, there is often eosinophilia. For rhabdomyolysis myalgia, the serum creatine kinase is significantly elevated.

A 44-year-old male presents to the emergency room with the "worst headache of his life." He is holding his head and appears severely distressed. This is a patient symptom that should prompt a clinician to look for signs of a: A. Migraine headache B. Subdural hematoma C. Subarachnoid haemorrhage D. Brain tumor

C. Subarachnoid haemorrhage The pain associated with subarachnoid hemorrhage is generally described as severe and acute in onset. The onset is often described as a thunderbolt or lightning. The severity is described as ―the worst headache of my life.‖ It is generally made worse by lying down. There is often associated nausea and/or vomiting and possible rapid deterioration in neurological function. Typical migraine pain begins unilaterally but may become generalized and may lateralize to the opposite side and/or radiate to the face or neck. The pain ranges from a dull ache to a throbbing or pulsatile pain. The pain is often severe and/or incapacitating and is often aggravated by movement, light, and noise. The headache associated with subdural hematoma is generally dull and aching in nature and may be transient. The history often includes a blow to the head, fall, or other injury, which preceded the pain. The pain will gradually worsen over days to weeks. Headache due to a brain tumor is difficult to diagnose. A headache that awakens a patient from sleep is often a brain tumor. Headache with neurological deficits should also raise suspicion of brain tumor.

True or False? During the physical exam, the physician will ask the patient to cough while the physician palpates the prostate gland.

False

True or False? Herpes simplex presents with a single chancre.

False

True or False? Meningitis may mimic the symptoms of rupture of an intracranial aneurysm or AVM.

False

True or False? Pain that is sharp and reproduced only when the patient raises the right arm suggests bursitis.

False

True or False? Strabismus may be a symptom of damage to the third cranial nerve.

False

True or False? The Barlow-Ortolani maneuver tests for dislocation of an adolescent's hip.

False

When obtaining the health history of a 60-year-old female, it is important to ask: A. The age of menopause B. About any bleeding since menopause C. Use of any hormone replacement treatment D. All of the above

D. All of the above Bleeding that occurs after menopause has been established is cause for concern. It may indicate endometrial cancer, and referral for endometrial biopsy or dilation and curettage (D&C) is warranted. If the patient is menopausal, ask about age of menopause, symptoms of menopause, and past or current use of hormone replacement therapy (HRT).

True or False? The descending RAS is essential for arousal from sleep.

False

True or False? The female urinary system contains two kidneys, one ureter, one bladder, and one urethra.

False

Which of the following is a common trigger of migraine headache? A. Missed meals B. Menses C. Alcohol D. All of the above

D. All of the above Common Migraine Triggers: - Stress - Caffeine - Altered sleep - Specific foods, missed meals - Menses - Alcohol - Hormone supplements

True or False? The nipple should be palpated while the patient is seated.

False

True or False? The wedged palpation technique is the most effective in detecting abnormalities.

False

A 55-year-old diabetic woman complains of a complete restriction of motion in her shoulder. What should be suspected? A) Myofascial injury B) AC joint separation C) Fractured clavicle D) Adhesive capsulitis

D) Adhesive capsulitis

What is true of the vas deferens? A) It is where sperm matures. B) It contains the cells of Leydig. C) It ends at the tip of the penis. D) Normally there are two.

D) Normally there are two.

Which of the following is true regarding pregnant patients? A) Increased estrogen and progesterone cause increased sensitivity in late pregnancy. B) Nipples and areolae become less pigmented. C) Breasts will feel softer and smoother on palpation as the pregnancy progresses. D) Palpation of the nipple may produce colostrum.

D) Palpation of the nipple may produce colostrum.

Which of the following is true? A) Electrical synapses are more common than chemical synapses. B) Synaptic transmission is an electrical response. C) Chemical synapses allow currents to travel in either direction. D) The release of neurotransmitters allows unidirectional conduction of an impulse.

D) The release of neurotransmitters allows unidirectional conduction of an impulse.

True or False? Urinary urgency/frequency, combined with dysuria, fever, and chills, indicates urethritis.

False

During a pelvic examination, a clinician asks the patient to bear down to inspect the vaginal outlet for: A. Cystocele B. Rectocele C. Prolapsed uterus D. All of the above

D. All of the above During a pelvic examination, the clinician can inspect the external genitalia for lesions, ulcerations, inflammation, warts, swelling, discharge, or nodules. Inspect the perineum and anal areas for fissures, hemorrhoids, inflammation, lesions, ulcers, warts, or nodules. Assess the support of the vaginal outlet by asking the patient to bear down while you look for bulging at the introitus, suggesting a cystocele, rectocele, or prolapsed uterus.

True or False? The epididymis feels softer than the testes.

True

True or False? The hand has two types of muscles and three major nerves.

True

True or False? The physician asks the patient to lie supine. The physician flexes the patient's neck and the patient involuntarily bends his knees. This is known as Brudzinski's sign.

True

True or False? The uterus is largely muscle.

True

What does routine urinalysis test for?

- pH - Specific gravity - Sodium level - Urine osmolality

Normal range for urinalysis urine osmolality

300-900 mOsm/kg of water

When palpating, what does localized warmth indicate? A) Blood leaking into the joints B) Synovial fluid leaking into the tissue C) Vascular constriction D) Infection

A) Blood leaking into the joints

A patient has trouble maintaining balance when walking normally, either with eyes open or closed. This suggests what? A) Cerebellar disease B) MS C) Romberg's test D) Guillain-Barre syndrome

A) Cerebellar disease

Cooper's ligaments are found in which kind of breast tissue? A) Fibrous B) Glandular C) Adipose D) Nervous

A) Fibrous

Which test assesses the kidneys, ureter, and bladder? A) IVP B) BMP C) Routine urinalysis D) 24-hour collection

A) IVP

Orchitis is likely to present as what? A) Pain in the genital region B) A thick, greenish discharge C) Nocturia D) Frequent urination

A) Pain in the genital region

Which of the following is a sexual dysfunction? A) Peyronie's disease B) Pyelonephritis C) Dysuria D) Prostatitis

A) Peyronie's disease

Which of the following assessments reflect the ability of the parasympathetic or sympathetic nervous system to innervate CN III? A) Pupillary function B) Decorticate posturing C) Verbal response D) Motor response

A) Pupillary function

Which of the following would be an abnormal finding when palpating the testes? A) Ridged B) Smooth C) Oval D) Rubbery

A) Ridged

Which of the following findings would indicate Nabothian cysts on a patient's cervix? A) Small, translucent, yellow nodules B) Columnar epithelium from endo-cervix is everted C) Bright red, soft protrusions D) Hard ulcers

A) Small, translucent, yellow nodules

A patient's os is slit-like in appearance. This indicates what? A) The patient has borne a child or children. B) The patient has not yet engaged in sexual activity. C) A pathology may be present. D) The patient is likely using hormone therapy.

A) The patient has borne a child or children.

Nicotine can contribute to which of the following? A) Urinary problems B) Sexual dysfunction C) Allergic vaginitis D) Amenorrhea

A) Urinary problems

A 78-year-old female comes to the clinic for a physical examination. She is accompanied by her daughter and looks to her daughter to answer questions during the interview. She was diagnosed with early Alzheimer's disease 2 years ago, and her daughter would like her current mental status evaluated. You ask the patient her daughter's name, and she answers correctly. You ask her the date and time, and she answers incorrectly. You hand the patient a pencil and ask her if she knows what it is. She replies with, "Is it a stick?" You ask the patient to put on a patient gown, and she does not know how to perform the task. These are examples of disorientation to time and: A. Agnosia and apraxia B. Anomia and aphasia C. Agnosia and ataxia D. Apathy and ataxia

A. Agnosia and apraxia Most organic dementias develop over months to years. There are typically no physical motor or sensory alterations until the condition is advanced. Memory impairment is the predominant symptom. There may be impairment in another area of cognitive functioning, such as with aphasia (producing language as well as understanding it), agnosia (perceptual impairment of environment), apraxia (inability to perform complex motor acts), and impairment in executive functioning (inability to plan, organize, sequence, and think abstractly). Ataxia is not a symptom of dementia— it is a problem with gait usually due to cerebellar dysfunction.

Which of the following may be a sign or symptom of prostate cancer? A) Body aches B) A rash C) Nausea D) Lower extremity edema

C) Nausea

Spina bifida is a disorder that is associated with a headache syndrome caused by: A. Chiari malformation B. Normal pressure hydrocephalus C. Meningioma D. Temporal arteritis

A. Chiari malformation Chiari malformations are brainstem malformations. There are three types of Chiari malfunction. Chiari type I is most often associated with occipital headaches and generally seen in the adult population but can be diagnosed at any age. Type I symptoms may be very vague and transient, and it is often misdiagnosed as another neurological disease. Type II is generally diagnosed in infants or children and is associated with myelomeningocele or other open neural tube defects or in adults with undiagnosed spina bifida occulta or tethered cord. Type III malformation is rare, diagnosed in infants, associated with cervical myelomeningocele or pseudomeningocele, and carries a very poor prognosis.

Which of the following infections often causes scarring of the fallopian tubes? A. Chlamydia trachomatis B. Pandida albicans C. Trichomonas vaginalis D. Gardnerella vaginalis

A. Chlamydia trachomatis Infection with N. gonorrhoeae can persist, leading to salpingitis, abscess, and peritonitis. As with infection caused by chlamydia trachomatis, long-term complications include scarring of the fallopian tubes, ectopic pregnancy, and infertility.

A major reason for the atrophic changes that occur in the pelvic floor muscles of the elderly woman is: A. Diminished levels of estrogen B. Increased levels of testosterone C. Autonomic neuropathy D. Decreased glycogen in the vagina

A. Diminished levels of estrogen The most significant age-related changes in women are related to decreased estrogen influence. As estrogen levels decline, the epithelium and supporting tissues of the pelvis atrophy, resulting in friable mucosa and possible prolapse of the pelvic structures. The change in relationship of the pelvic structures results in hypermobility of the bladder base, pelvic muscle weakness, and urethral weakness, resulting in stress UI in women. The decreased glycogen content of the vaginal epithelium causes decreased lactic acid metabolism by Döderlein's bacillus, an increased pH of vaginal secretions, and, therefore, an increased risk for UTI.

An 85-year-old female patient arrives by ambulance to the emergency department accompanied by her husband. He reports that his wife had been ill with pneumonia and, 2 days ago, went to the family physician who prescribed azithromycin twice a day. The husband reports that he is making sure she gets the medicine. His wife has been staying in bed and resting. She awoke from sleep last night and was extremely agitated, left the house, and was walking outside. She did not recognize her husband and wanted to call the police. Which of the following is an appropriate question for the history? A. Does your wife have dementia or frequent episodes of confusion? B. Has your wife been running a fever? C. Is your wife allergic to any medication? D. Has your wife ever had a mental status exam?

A. Does your wife have dementia or frequent episodes of confusion? Delirium can be observed in both elderly and younger patients and is generally defined as an acute confusional state, affecting all aspects of cognition and mentation. The signs and symptoms of delirium generally have a more acute or rapidly progressive onset as opposed to the slow, gradual decline noted in the organic dementias. The acute mental status change is often associated with other signs or symptoms—such as hallucinations, illusions, incoherent speech, and constant aimless activity—that help to narrow the differential diagnosis. Electrolyte disturbances, infection, and polypharmacy are frequent causes of delirium in the elderly.

A 52-year-old female complains of discharge from one of her breasts. There is no pain, no mass, and no skin changes. The physical examination is normal. When putting pressure on the affected breast, the nipple expresses a small amount of thick, white discharge. These findings are consistent with: A. Duct ectasia B. Mastitis C. Fibroadenoma D. Fibrocystic breasts

A. Duct ectasia Discharge from one breast is commonly benign and frequently caused by duct ectasia or an intraductal papilloma. Duct ectasia results in dilation of one major breast duct and causes approximately one-third of the cases of pathological discharge. Papillomas are responsible for 44% of pathologic discharge. Because the discharge associated with ductal ectasia is often stagnant, the discharge is cheesy in appearance. It is often associated with noncyclic breast discomfort and a subareolar lump at the site of the dilated duct. Because one major duct is involved, the discharge comes also from one duct or nipple area.

Which of the following infectious diseases is often the cause of a reactive arthritis? A. Gonococcus infection B. Beta-hemolytic streptococcus infection C. Chlamydia infection D. Norovirus infection

A. Gonococcus infection Gonorrhea is a sexually transmitted infection that often involves reactive arthritis. The polyarthritis is often migratory and affects lower extremities as well as the hands. In addition to the arthritis, the syndrome usually includes a nonpruritic dermatitis and tenosynovitis. Generalized muscle aches and fever are also common.

Your patient is a 55-year-old male who presents with sudden, severely tender, swollen, erythematous elbow. The patient reports that he experienced similar symptoms in the past. You note the right elbow has a swollen, tender, soft 3 cm round nodule. The clinician should recognize these are signs and symptoms of: A. Gouty arthritis B. Rheumatoid arthritis C. Epicondylitis D. Reiter's syndrome

A. Gouty arthritis Gout is a classic cause of monoarthritis, and most cases involve a joint in the lower extremities. Acute pain usually develops in one joint, with swelling, redness, and warmth, and the severity of the pain increases rapidly. ROM of the affected joint(s) is limited by pain, and there is significant tenderness to the site. Patients who have had gout for an extended time often have gouty tophi, which are soft tissue nodules containing urate crystals. The olecranon bursa is a common site for tophi development, and the tophi are often painful.

Muscle strength is assessed by: A. Having the patient move their muscle against the clinician's resistance B. Examining how the patient performs active range of motion C. Passively stretching the patient's muscle D. Performing passive range of motion on the patient's muscle

A. Having the patient move their muscle against the clinician's resistance Muscle strength is determined by asking the patient either to resist the examiner's attempt to flex or extend a muscle group or to flex or extend the muscles against the examiner's resistance. Muscle strength is graded 0 (no evidence of strength) to 5 (complete or full resistance). Pain, contracture, and disease can all affect muscle strength.

It is important for the clinician to ask the female patient about multiple sexual partners. It is known that multiple sexual partners increases patient risk for: A. Human papilloma virus infection B. Endometriosis C. Uterine cancer D. Ovarian cysts

A. Human papilloma virus infection The matter of sexual activity and sexual partners is an important area to inquire about, especially with the high risk of HIV and other STDs. Multiple sexual partners, even in the absence of STDs, puts women at risk for infection with human papilloma virus (HPV).

The most common complication of an untreated urinary obstruction due to a ureteral calculus is: A. Hydronephrosis B. Renal artery stenosis C. Ureteral rupture D. Kidney mass

A. Hydronephrosis Hydronephrosis is swelling of the renal pelvis caused by an obstructing stone, ureteral stricture, prostatic hyperplasia, or renal or abdominal tumor that prevents the kidney from draining. The obstruction can be unilateral or bilateral, symptoms can be sudden or gradual in onset, and progressive renal damage will occur with time.

An 18-year-old female patient presents with repeated urinary tract infections. She has no risk factors in her history, and her physical examination is unremarkable. She also has a normal pelvic exam. Which of the following should be obtained if anatomic abnormalities are suspected? A. Intravenous pyelogram B. Ultrasound of the kidneys C. Cystoscopy D. Transvaginal ultrasound of the bladder

A. Intravenous pyelogram Risk factors for the development of a UTI include increasing age, recent sexual intercourse, a history of UTI, use of a diaphragm or cervical cap, and anatomic abnormalities. Urine cultures should be considered in the patient with recurrent UTI or refractory UTI and in the patient who appears toxic. Blood for CBC and electrolyte tests should also be drawn based on the overall clinical presentation. If anatomic causes are suspected, diagnostic imaging, such as KUB or IVP, should be arranged based on the suspected cause and patient's presentation.

Which of the following describes the pathology of De Quervain's tenosynovitis? A. Irritation of a tendon located on the radial side of the wrist, near the thumb. B. Impingement of the median nerve, causing pain in the palm and fingers C. Fluid-filled cyst that typically develops adjacent to a tendon sheath in the wrist D. Ulnar nerve compression at the olecranon process

A. Irritation of a tendon located on the radial side of the wrist, near the thumb. De Quervain's tenosynovitis involves irritation of a tendon located on the radial side of the wrist, near the thumb. With overuse, the tissues surrounding the tendon sheath hypertrophy, causing pressure on the tendon and making it difficult to move. The pain is usually limited to the radial aspect of the wrist and area immediately around the base of the thumb. Pain increases with use of the hand, such as with gripping maneuvers. Other symptoms include swelling, decreased sensation, and limited ROM with a locking sensation with thumb motion. The Finkelstein maneuver (Fig. 14.7) is used to diagnose De Quervain's disease. A positive test results in pain, which is often severe. Patients who can repeatedly open and close the fist with smooth thumb motion are unlikely to have De Quervain's.

A 17-year-old male complains of severe right knee pain. He was playing football when he heard a "pop" at the moment of being tackled and his knee "gave away" from under him. On physical examination, there is right knee swelling and decreased range of motion. There is a positive anterior drawer sign. These findings indicate: A. Knee ligament injury B. Osgood-Schlatter disease C. Prepatellar bursitis D. Chondromalacia patella

A. Knee ligament injury The anterior, medial, and lateral knee ligaments are vulnerable to injury in athletic activities. The mechanism through which the anterior cruciate ligament (ACL) is typically injured involves deceleration combined with sudden turning or pivoting. The medial collateral ligament (MCL) is most prone to injury through motions that place valgus stress on the knee. Compared with ACL and MCL injury, damage to the lateral collateral ligament (LCL) is much rarer but typically occurs when sudden varus stress is placed on the knee. The patient often relates history of an acute trauma followed by the onset of pain, swelling, and limited mobility. Often patients recall hearing or feeling a "pop" at the moment of injury and/or "give-away" sense. ACL injury is identified through a positive drawer (Fig. 14.9) and/or Lachman's test (Fig. 14.10). Laxity of the LCL is assessed by placing varus stress on the knee with the leg both extended and flexed.

A 65-year-old male patient complains of frequency, urgency, hesitancy, and weak urine stream. Physical examination is unremarkable except DRE reveals an enlarged, firm, non-tender prostate gland. Urine culture reveals 100 bacteria CFU/mL. The clinician should suspect: A. Lower urinary tract infection due to benign prostatic hyperplasia B. Bacteriuria due to prostatitis C. Obstructive uropathy due to prostate cancer D. Upper urinary tract infection due to benign prostatic hyperplasia

A. Lower urinary tract infection due to benign prostatic hyperplasia Urinary tract infection is uncommon in males and when UTI is diagnosed in the male the etiology should be determined. Benign prostatic hyperplasia presents as an enlarged, firm, non-tender prostate on DRE. An enlarged prostate is commonly diagnosed when a male presents with a lower urinary tract infection. The enlarged prostate causes urine retention in the bladder, and stagnant urine in the bladder is a medium for bacterial growth causing lower UTI.

A 33-year-old male marathon runner presents with knee pain. Which of the following tests is positive if the meniscus of the knee is torn? A. McMurray's test B. Straight leg raising sign C. Anterior drawer sign D. Lachman test

A. McMurray's test Tears or disruptions of the meniscus sheath of cartilage are associated with OA in older persons and with athletic activities in younger persons. There is typically a sudden onset of pain and swelling over the lateral or medial joint line as well as locking and painful popping. Onset often follows a twisting injury. Point tenderness is present over the joint line, with mild effusion. A positive McMurray's test is often present.

Your patient that has a uterine fibroid complains of bleeding through up to 10 tampons per day. Excessive bleeding at the time of the menstrual cycle is termed: A. Menorrhagia B. Metrorrhagia C. Dysmenorrhea D. Premenstrual syndrome

A. Menorrhagia The menstrual history includes any episodes of amenorrhea, menorrhagia (excessive bleeding at the time of the menstrual cycle), metrorrhagia (bleeding at irregular noncyclic intervals), dysmenorrhea, and postmenopausal bleeding.

When examining the breasts of a 45-year-old female patient, you note nodularity throughout both breasts. There is no dominant mass, tenderness, nipple change, or skin change. Which of the following condition is most likely? A. Multiple fibroadenomas B. Fibrocystic breasts C. Mastalgia D. Mastitis

A. Multiple fibroadenomas Breast tissue is normally glandular and may have a rather nodular consistency. The degree of nodularity tends to fluctuate through the menstrual cycle in premenopausal women. A dominant breast mass is a mass that persists throughout a woman's hormonal cycles, is larger and firmer than any other irregularities, and differs from rest of the breast tissue. A fibroadenoma is a soft discrete breast mass that is moveable, non-tender, and benign. Mastalgia is a condition of pain in the breast. Mastitis is an infection that causes erythema, swelling, and discharge from the breast.

A 3-year-old male toddler complains of sudden arm pain. The mother indicated that pain occurred suddenly while his 9-year-old sister was helping him get dressed. The child presents with the arm flexed while protecting his elbow. On physical examination, there is tenderness along the radius with no swelling or evidence of trauma. This is a typical history of: A. Nursemaid's elbow B. Epicondylitis C. Smith fracture D. Nursery pseudo-fracture

A. Nursemaid's elbow Also known as pulled elbow or toddler's elbow, nursemaid's elbow involves the head of the radius slipping under the annular ligament in children, usually between 1 and 4 years of age. The condition occurs when traction is applied to the young child's hand or wrist. There is a history of sudden onset of pain associated with sudden immobility of the affected arm as the child protects the elbow. The parent may be able to identify a situation in which the child's hand was held and traction applied. The child may have moved in an opposite direction or injury could occur while pulling the arm through clothing. There is no associated swelling or inflammation. Examination is otherwise normal with the exception of resistance to attempts to move the arm, elbow, and, possibly, wrist. There may be tenderness along the upper margin of the radius.

Your patient is a 36-year-old woman who complains of milky discharge from both her breasts, episodes of headache, and menstrual irregularity. The patient is on no medications. Upon examination, there are no significant findings, and the patient is not pregnant, breastfeeding, or in the postpartum period. It is important to: A. Order prolactin level B. Obtain an MRI of head C. Have the patient keep a menstrual cycle diary D. Refer the patient for mammogram

A. Order prolactin level Galactorrhea can be a symptom of endocrine disorder, particularly pituitary dysfunction. In prolactinoma of the pituitary, a patient complains of galactorrhea, headaches, vision change, relative infertility, and amenorrhea. Laboratory studies should include a pregnancy test, prolactin level, and thyroid functions.

A 60-year-old female patient complains of pain in the hands that is worse in the morning. On physical examination, the thumb metacarpophalangeal joint is swollen on both hands. There is swelling of the proximal and distal interphalangeal joints bilaterally. These are typical signs of: A. Osteoarthritis B. Rheumatoid arthritis C. Normal aging D. Gouty arthritis

A. Osteoarthritis Compared with RA, OA has a higher likelihood of affecting larger joints, such as the hips and knees. Like RA, OA also frequently involves the small joints of the hands, although it tends to occur at the distal interphalangeal joints (Heberden's nodes) and proximal interphalangeal joints (Bouchard's nodes). Most frequently, the second and/or third digits and the base of the thumb are involved. The distribution is asymmetrical. The pain and stiffness associated with OA often improve with moderate use and are worse after extended periods of rest. If three or more metacarpophalangeal joints are swollen, the differential should include RA.

What does balantitis often present with? A. Pain and prepuce swelling B. Edema, dysuria, and pruritis C. Scrotal pain, edema, possible fever

A. Pain and prepuce swelling

In the health history, you ask a young adult female patient about safe sex and the use of contraception. The patient does not understand what you mean by safe sex. This is an indication for: A. Patient teaching and counseling B. Promotion of oral contraceptive use C. Pregnancy testing and counseling D. STD, HIV, and hepatitis testing

A. Patient teaching and counseling Ask the patient about the type of birth control being used, if any. Ask about condom use. Be sure to inquire as to the consistency with which the patient uses birth control methods. Often patients deny the use of birth control, deny the desire for pregnancy, and yet admit to being sexually active. This definitely indicates the need for health teaching and counseling.

Pap smears assist in early diagnosis of uterine cancer. The peak incidence of endometrial cancer occurs in: A. Postmenopausal women aged 50 to 70 years old B. Postmenopausal women aged 40 to 50 years old C. Premenopausal women between 30 and 40 years old D. Premenopausal women between 40 and 50 years old

A. Postmenopausal women aged 50 to 70 years old It is important to perform Pap smears on older women because 25% of cervical cancers occur in women older than 65 years of age, and the peak incidence of endometrial cancer occurs in postmenopausal women between 50 and 70 years. In patients who have had hysterectomies, a smear of the vaginal cuff is still suggested as well as inspection of the vaginal mucosa and external genitalia looking for signs of malignancy.

If amenorrhea is reported by the female patient, the clinician should attribute this to proven otherwise. A. Pregnancy B. Ovarian failure C. Lack of progesterone D. Menopause

A. Pregnancy Last menstrual period is one of the most important questions to ask, particularly when prescribing medications, because many are contraindicated in pregnancy. If menstrual cycles are not regular, pregnancy should be ruled out first, and then other diagnoses can be considered. The menstrual history includes any episodes of amenorrhea, menorrhagia (excessive bleeding at the time of the menstrual cycle), metrorrhagia (bleeding at irregular noncyclic intervals), dysmenorrhea, and postmenopausal bleeding. Amenorrhea has many causes, including pregnancy; anorexia nervosa; excessive exercise; low body fat; and disorders or tumors of the hypothalamus, pituitary gland, ovary, uterus, and thyroid gland.

A 27-year-old male comes in to the clinic for symptoms of dysuria, urinary frequency, as well as urgency and perineal pain. Transrectal palpation of the prostate reveals a very tender, boggy, swollen prostate. The clinician should recognize these as signs of: A. Prostatitis B. Prostate cancer C. Urethritis D. Benign prostatic hyperplasia

A. Prostatitis Prostatitis is an acute or chronic infection of the prostate gland. Acute bacterial prostatitis is usually the result of infection by aerobic gram-negative rods (coliform bacteria or Pseudomonas). Enterococcus faecalis, an aerobic gram-positive bacteria, can also cause prostatitis. Routes of infection are ascent from the urethra, reflux of infected urine into the prostatic ducts, direct extension of bacteria, and migration via the lymphatic and vascular system. Acute symptoms commonly include fever, low back and perineal pain, possible penis pain, urinary urgency and frequency, nocturia, dysuria, and muscle and joint aches. Transrectal palpation of the prostate reveals a very tender, boggy, swollen prostate.

Primary dysmenorrhea is pain with menses that occurs due to: A. Rise in prostaglandins B. Endometriosis C. Fibroid tumor D. Excessive estrogen

A. Rise in prostaglandins Dysmenorrhea is the most common gynecological complaint, particularly in the adolescent and young adult population. Primary dysmenorrhea is due to a rise in prostaglandins that occurs at the onset of menses, and it has been found that prostaglandins are higher in women with dysmenorrhea. Recently, increased leukotriene levels have been found to contribute to dysmenorrhea. Other psychosocial variables may contribute, such as response to pain, anxiety, stress, and attitudes about menstruation. Secondary dysmenorrhea is most often caused by endometriosis. Other causes include chronic PID, adhesions, IUDs, cervical stenosis, and uterine fibroids.

Your 33-year-old female patient reports the development of a "cold sore" on her lower lip. She asks if this is the kind of sore that is transmitted during sex. The clinician should explain that: A. The sore can be either herpes type 1 or herpes type 2, and both can be transmitted during sex B. A cold sore in the oral region is caused by herpes type I, and this is not contagious C. Herpetic lesions on the lips and in the mouth are herpes type II and caused by oral sex D. A blood test is done to determine the specific type of virus because treatments for I and II differ

A. The sore can be either herpes type 1 or herpes type 2, and both can be transmitted during sex The majority of genital herpes infections are due to HSV2, and approximately 10% are due to the HSV1 virus that has been transferred to the genital area either by sexual or oral contact. Although blood tests can be done to determine the presence or absence of antibodies to HSV1 and HSV2, it is mostly academic because the symptoms, transmission, and treatment are the same regardless of type.

A 46-year-old administrative assistant complains of pain in the wrist that radiates into the palm and into the fingers. The clinician can test the patient for carpal tunnel syndrome by eliciting which of the following signs? A. Tinel's sign B. Apley's sign C. Finkelstein sign D. Lhermitte's sign

A. Tinel's sign Carpal tunnel syndrome causes a range of neurological symptoms, including pain, paresthesia, and weakness. Frequently, nighttime pain is an early symptom. There may be a swelling at the wrist related to inactivity or flexion at night. The pain and/or paresthesias typically involve the anterior aspects of wrist, medial palm, and first three digits on the affected hand. However, pain may radiate up the forearm to the shoulder with numbness and tingling along the median nerve. Over time, hand weakness often develops. Pain and paresthesia are often relieved by the patient "shaking" the affected hand in a downward fashion; this is called the flicking sign. A positive Tinel's sign is elicited by tapping on the median nerve at the carpal tunnel, causing pain and tingling along the median nerve. Phalen's maneuver reproduces the pain after 1 minute of wrist flexion against resistance.

To diagnose ovarian disorders, which of the following diagnostic studies is most helpful? A. Transvaginal ultrasound B. Pelvic CT scan C. Pelvic MRI scan D. Pap smear

A. Transvaginal ultrasound Pelvic examination has been shown to have a low sensitivity and specificity for ovarian cancer, with many tumors smaller than 10 cm being missed. Several diagnostic studies are useful for detecting ovarian cancer. The CA-125 blood test by immunoassay is used as a tumor marker for treatment decisions but can also be used as a diagnostic tool. False-positive elevations in CA-125 do occur with endometriosis and pelvic surgeries; therefore, tests should be interpreted with caution and followed up with transvaginal ultrasound. Transvaginal ultrasound will show a solid, irregular mass that may be adhered to adjoining structures. A Pap smear may contain malignant cells. X-rays may show metastatic lung or bone lesions. Transvaginal ultrasound will give the best information for differentiating a blood- or fluid-filled ovarian cyst from a solid mass.

Which of the following findings on the bimanual exam of a female patient in early pregnancy requires further investigation? A. Unilateral adnexal tenderness B. Bluish colored cervix C. Varicose veins of vaginal walls D. None of the above

A. Unilateral adnexal tenderness On bimanual examination, significant unilateral adnexal pain or tenderness in an early pregnant female can indicate ectopic pregnancy.

A 33-year-old female patient complains of white, thick vaginal discharge and severe pruritus of the outer genitalia. On pelvic exam, the vaginal introitus is red and swollen with a thin layer of white exudate. The bimanual exam is normal. A KOH prep is positive for hyphae. These findings are indicative of: A. Vaginal candidiasis B. Trichomonas infection C. Chlamydia D. Bacterial vaginosis

A. Vaginal candidiasis The discharge associated with yeast vaginitis can be differentiated from other infections by symptomatology and wet prep examination. The discharge is very thick, curdlike, and adheres to the vaginal walls. Intense vulvar itching accompanies the discharge. In most cases, inflammation and swelling around the labia and introitus occur. A high suspicion of yeast can be made on physical examination with a wet prep to confirm the diagnosis of Candida albicans. A 10% KOH (potassium hydroxide) prep is most helpful for visualizing budding yeast and hyphae microscopically.

A 34-year-old female who gave birth 2 weeks ago developed fever and pain in the right breast while nursing her baby. The patient has tenderness, eythema, and swelling of the nipple on the right breast. Which of the following diagnostic studies is indicated? A. White blood cell (WBC) count B. Breast milk culture C. Ultrasound D. Fine needle biopsy

A. White blood cell (WBC) count For mastitis, white blood count should be obtained and is usually elevated. Even though the breast milk can be cultured, this is not generally recommended. If the presentation is atypical—that is, the patient is not lactating—and there are no associated systemic signs or symptoms, a consultation should be obtained and mammography ordered to determine the definitive diagnosis and rule out malignancy.

Which of the following is one of the "Five Ps" of musculoskeletal examination? A) Petrification B) Problem C) Pulselessness D) Position

C) Pulselessness

Stimulating a newborn's lips to test deviation of the head toward the stimulus tests what? A) Babinski B) Placing C) Rooting D) Stepping

C) Rooting

To percuss the kidneys, the physician must strike what? A) The base of the spine B) The soft tissue of the abdomen C) The costovertebral angle D) The vertebral column

C) The costovertebral angle

Injury to which part of the body does the text explicitly state carries a high likelihood of damage to nerves and/or blood vessels? A) Knee B) Shoulder C) Elbow D) Hip/pelvis

C) Elbow

A patient's pupils are not the correct size for the amount of light. Damage to which cranial nerve is suspected? A) I B) VI C) III D) II

C) III

Of the following, which cranial nerves are only of motor type, not sensory? A) I, VII, and X B) IV, V, and VI C) III, XI, XII D) I, VIII, IX

C) III, XI, XII

When assessing range of motion, a patient's ulnar deviation and radial deviation should be __________, respectively. A) 20 degrees and 55 degrees B) 55 degrees and 20 degrees C) 70 degrees and 70 degrees D) 25 degrees and 40 degrees

B) 55 degrees and 20 degrees

Which of the following is true regarding nipple discharge? A) Discharge emanating from multiple ducts suggests cancer. B) A bloody discharge suggests intraductal papilloma. C) Oral contraceptives and steroids can cause a greenish discharge. D) Duct ectasia produces a clear or milky discharge.

B) A bloody discharge suggests intraductal papilloma.

What it the normal capacity of the urinary bladder? A. 700 mL B. 400 mL C. 200 mL D. 100 mL

B. 400 mL The bladder has several functional features that contribute to normal continence: its normal capacity of 400 to 500 mL; the fullness sensation; the ability to accommodate various volumes without changes in intraluminal pressure; the ability to initiate and sustain contraction until the bladder is empty; and a response to the voluntary inhibition of voiding despite the inherent involuntary nature of the organ.

A 55-year-old woman complains of amenorrhea and hot flashes for the last 3 months. Pregnancy test is negative. The clinician should recognize that menopause is defined as the absence of menses for: A. 3 months B. 6 months C. 1 year D. 18 months

B. 6 months Menopause is the absence of menses for at least 6 months. Age of menopause varies greatly, although age 50 to 55 is the typical range of onset. Although the absence of menses in a woman around the age of 50 years is diagnostic for menopause, measurement of FSH, LH, and estradiol levels are helpful in confirming the diagnosis.

What does urethritis often present with? A. Pain and prepuce swelling B. Edema, dysuria, and pruritis C. Scrotal pain, edema, possible fever

B. Edema, dysuria, and pruritis

A 23-year-old female complains of a white-yellow vaginal discharge with a fishy odor for the last 2 days. A wet prep shows characteristic clue cells (epithelial cells embedded with bacteria). These findings are indicative of: A. Chlamydia infection B. Bacterial vaginosis C. Vaginal candida D. Gonorrhea

B. Bacterial vaginosis Gardnerella vaginalis causes bacterial vaginosis, the most prevalent vaginal infection. Of those women who are symptomatic, the overwhelming complaint is of a malodorous discharge. The odor of G. vaginalis is a distinct, fishy odor. The odor is usually noticeable during the pelvic examination, but a few drops of 10% KOH solution on the wet prep slide augments the odor. The discharge is fairly thick and white. Patients do not complain of itching, and, generally, there is no inflammation of the vaginal mucosa. Microscopically, the wet prep shows the characteristic clue cells (epithelial cells embedded with bacteria).

A 77-year-old male is brought into the emergency department accompanied by his daughter. She reports that her father has been complaining of right-handed weakness for 2 to 3 hours and that she has noticed him slurring his speech. On physical examination, the right hand grip strength is 1/ 5 compared to the left hand grip of 5/5. A facial droop is noted on the right side of the face. Cranial nerve dysfunction is noted in the right-sided CNVII, CN V, CNIX, CNX, CNXI, and CNXII. The clinician should request the following diagnostic test immediately: A. MRI of head B. CT of the head C. Lumbar puncture D. ECG

B. CT of the head Strokes are divided into two main categories: thrombotic and hemorrhagic; however, the two can be difficult to differentiate using clinical signs and symptoms. The onset is usually an abrupt altered level of consciousness accompanied by hemiparesis or hemiplegia. Patients may experience confusion, memory impairment, and aphasia. Signs and symptoms vary with the location and severity of the stroke. Mentation and cognitive changes may be temporary or permanent depending on the extent of injury. Communication alterations stemming from fluent or receptive aphasia may be mistaken as dementia. A CT scan, without contrast, is the preferred imaging study in early stroke because hemorrhage may be difficult to determine on an MRI in the first 48 hours. In studies of ischemic stroke patients, researchers have shown the reversibility of abnormalities on CT or MRI through the use of thrombolytic therapy within a 3-hour window.

Which of the following infections can cause an eye infection in the newborn during vaginal delivery? A. Candida vaginitis B. Chlamydia trachomatis C. Trichomonas vaginalis D. Gardnerella vaginalis

B. Chlamydia trachomatis Chlamydia is an STD caused by the bacteria Chlamydia trachomatis. It is thought to be the main cause of salpingitis and scarring of the fallopian tubes, thus leading to ectopic pregnancy or infertility. It can also cause conjunctivitis in the neonate if present during delivery. Patients are often asymptomatic with chlamydia infections but may present with mucopurulent discharge, dysuria, abdominal pain, fever, and abnormal vaginal bleeding. Cervicitis and cervical motion tenderness on physical examination indicate PID. It is the causative organism in the majority of nongonococcal urethritis in both women and men.

Which of the following organisms can cause a syndrome that includes arthritis and conjunctivitis? A. Neisseria gonorrhea B. Chlamydia trachomatis C. Trichomonas vaginalis D. Gardnerella vaginalis

B. Chlamydia trachomatis Reiter's syndrome, a serious systemic complication of chlamydia infection, occurs more commonly in men and is characterized by urethritis, arthritis, and conjunctivitis or uveitis.

A 41-year-old male patient presents to the emergency department complaining of severe headache pain. He describes it as a piercing, right sided head pain that occurred earlier in day for about 1 hour and now is recurring. You note lacrimation and rhinorrhea on the right side of the face. Which of the following types of headache is the patient describing? A. Sinusitis B. Cluster headache C. Migraine D. Subdural hematoma

B. Cluster headache Cluster headaches have rapid crescendo patterns, peaking in approximately 10 to 15 minutes and often lasting 30 to 60 minutes per episode (rarely lasting over 2 hours each). Attacks occur as frequently as two to three times per day. The pain is generally in the area of the trigeminal nerve and is described as unilateral, penetrating, sharp, excruciating, and unrelenting in nature. There may be associated unilateral lacrimation, nasal congestion or rhinorrhea, pallor, flushing, conjunctival redness, ptosis—all on the same side as the pain.

A 65-year-old female complains of frequency and urgency of urination and episodes of urinary incontinence. On pelvic examination, there is smooth, soft bulge of the anterior vaginal wall that is more pronounced with straining. These findings are indicative of: A. Rectocele B. Cystocele C. Uterine prolapse D. Anteflexed uterus

B. Cystocele A cystocele can contribute to urinary frequency, urgency, and infection, and stress incontinence. The bimanual examination reveals a smooth, soft bulge of the anterior vaginal wall that is more pronounced with straining. Kegel exercises may be helpful, but surgical repair may be needed.

A 77-year-old female patient with heart failure complains of frequency and urgency of urination during the day and nocturia of 2 to 3 times a night. Which of the following medications can cause these symptoms? A. Beta adrenergic blocker B. Diuretic C. ACE inhibitor D. Digitalis

B. Diuretic A patient may complain of frequency that she or he associates with a particular medication. Alternatively, this association may not be clear until a complete list of the patient's medications is available. Signs and symptoms are primarily frequency and urgency, both of which may be complicated by coexisting GU conditions, such as BPH, some type of incontinence, or mobility issues. A review of a patient's medications and administration schedule should be sufficient to determine that a diuretic is a contributing cause. Consider urinalysis to rule out a UTI.

A 19-year-old female complains of sudden severe left-sided lower quadrant abdominal pain as well as nausea and vomiting for the last 2 weeks. Last menstrual period was 2 months ago. On physical examination, there is left sided lower quadrant abdominal tenderness and heel strike is positive. Pelvic examination reveals bluish discoloration of the cervix and left-sided adnexal tenderness. These signs and symptoms are consistent with: A. Ovarian cyst B. Ectopic pregnancy C. Pelvic inflammatory disease D. Follicular cyst

B. Ectopic pregnancy These are key signs of ectopic pregnancy. Amenorrhea in a female of childbearing age should be treated as pregnancy until proven otherwise. Sudden pain is characteristic of tubal ectopic pregnancy, when the embryo outgrows the space within the fallopian tube. The nausea and vomiting are symptoms of early pregnancy. Left-sided lower quadrant pain, bluish discoloration of the cervix, and left-sided adnexal tenderness are all signs of tubal pregnancy. Positive heel strike indicates peritoneal irritation.

Postmenopausal vaginal bleeding commonly indicates: A. Hormonal imbalance B. Endometrial cancer C. Endometriosis D. Atrophic vaginitis

B. Endometrial cancer Abnormal uterine bleeding is much more common in the younger population, especially during the teen years when menstrual patterns are becoming established. Also, during the early reproductive years, malignancies are much less likely to be the cause. Most cases of dysfunctional uterine bleeding are due to organic causes and to dysfunction of the hypothalamic-pituitary-ovarian axis. Bleeding after menopause has been established is cause for concern, and referral for endometrial biopsy is a must.

Your 24-year-old female patient comes to the clinic for a routine pelvic exam and Pap smear. It is recommended that a female of this age obtain a Pap smear: A. Every year B. Every 3 years C. Every 5 years D. None of the above

B. Every 3 years In March 2012, updated screening guidelines were released by the U.S. Preventive Services Task Force, American Cancer Society, American Society for Colposcopy and Cervical Pathology, and American Society for Clinical Pathology. These guidelines recommend a Pap smear by age 21 and a Pap smear every 3 years for women ages 21 to 29. Women ages 30 to 65 should be screened every 5 years with Pap and HPV co-testing or every 3 years with Pap smear alone. Recommendations for women with a history of abnormal Pap smears should be adjusted as needed for follow-up (National Cancer Institute, 2012).

A 65-year-old male complains of a headache that feels "like a knife is cutting into his head." He also reports feeling right-sided scalp and facial pain and "seeing double" at times. He has a history of hypertension and hyperlipidemia. His medications include beta blocker, statin drug, and an ACE inhibitor. On physical examination, you note palpable tenderness over the right side of the forehead. There are no neurological deficits. Vision is 20/20 with lenses. No weakness of extremities. CN II to XII are intact. The history corresponds to which of the following disorders? A. Drug toxicity B. Giant cell arteritis C. Cluster headache D. Migraine headache

B. Giant cell arteritis Temporal arteritis is also referred to as giant cell arteritis or cranial arteritis. It is characterized by chronic inflammation and the presence of giant cells in large arteries, usually the temporal artery, but can occur in the cranial arteries, the aorta, and coronary and peripheral arteries. It affects the arteries containing elastic tissue, resulting in narrowing and eventual occlusion of the lumen. It occurs more among persons over 50 years of age and is slightly more common in females than in males. The cause is unknown, but there seems to be a genetic predisposition. If left untreated, arteritis can rapidly lead to blindness that is often irreversible. The most common chief complaint is head pain that is lancinating, sharp, or "ice pick" in nature. Patients often complain of visual changes, including amaurosis, diplopia, blurred vision, visual field cuts, eye pain, periorbital edema, and intermittent unilateral blindness. Other common presenting symptoms include scalp and/or jaw tenderness, facial pain, and tenderness to palpation over the affected artery. The pain is generally hemicranial but can be bilateral or diffuse. There may be eye pain, which is usually bilateral; periorbital edema may be present. Other potential associated symptoms include an intermittent fever (generally low grade), nausea, and/or weight loss.

Which of the following microorganisms is a common cause of meningitis? A. Enterococcus B. Haemophilus influenza C. Influenza virus D. Chlamydia

B. Haemophilus influenza Meningitis involves inflammatory central nervous system (CNS) disease generally caused by either viral or bacterial infection. The etiology of meningitis includes community-acquired, post- traumatic, aseptic, carcinomatous, or transferred from another bodily source. The most common organisms belong to such genera as Streptococcus, Neisseria (meningitides), Haemophilus (influenzae), Listeria, Staphylococcus (aureus) as well as gram-negative bacilli and gram-positive cocci. Meningitis can affect persons of all ages, including children.

A 43-year-old male presents to the emergency department in a stupor accompanied by his wife. She called the ambulance because her husband was difficult to awaken this morning. She reports that for the last 3 days, he has had fever and upper respiratory infection. Yesterday he had a headache. He has had no medical treatment. On physical examination, the patient demonstrates 103 fever, sluggish pupil response, and nuchal rigidity. Which of the following should the clinician attempt to elicit next? A. Hallpike maneuver B. Kernig's sign C. Epley maneuver D. Babinski's sign

B. Kernig's sign In meningitis fever, photophobia, phonophobia, nausea, vomiting, and nuchal rigidity occur. Patients can rapidly decline to delirium, seizures, and, if untreated, coma. On neurological examination, the patient may be lethargic and febrile and have altered mentation along with nuchal rigidity and/or guarding, contracted and sluggish pupils, and a generally "toxic" appearance. Brudzinski's and Kernig's signs are helpful in assessing potential meningeal conditions (see Box 15.6). Delirium or acute confusion necessitates immediate transfer to an emergency department for treatment, as the patient can rapidly deteriorate to coma.

Your 24-year-old female patient complains of dysuria as well as frequency and urgency of urination that develops the day after she uses her diaphragm. Urine culture reveals a bacterial count of 100 CFU/mL. These signs and symptoms indicate: A. Upper urinary tract infection B. Lower urinary tract infection C. Normal bacteriuria D. Urethritis

B. Lower urinary tract infection In women whose symptoms suggest uncomplicated UTI, a culture of greater than 102 CFU/mL of a specific bacterium is indicative of cystitis. However, this CFU number is controversial. Infectious disease specialists recommend use of 103 CFU / mL as diagnostic of UTI. In the past, a bacterial count of 105 CFU/mL was regarded as significant for UTI. UTI diagnosis relies on the patient's subjective report of symptoms of UTI, not only the number of bacteria. Patients with asymptomatic bacteriuria can have bacterial counts as high as 10 5 CFU/mL. Asymptomatic bacteriuria is not treated. Risk factors for the development of a UTI include increasing age, recent sexual intercourse, a history of UTI, use of a diaphragm or cervical cap, and anatomic abnormalities.

Bloody breast discharge is associated with which of the following? A. Bleeding disorder such as von Willebrand disease B. Malignancy C. Excessive non-steroidal anti-inflammatories D. A and C

B. Malignancy Bloody discharge is often associated with malignancy but can stem from other conditions. If blood is not evident, the discharge should be tested for blood, using guaiac process.

A 45-year-old female patient presents in the emergency department due to multiple chest injuries as a result of a motor vehicle accident. There is a palpable, tender, irregular 3 cm soft mass located in the left breast, with erythema and swelling of the breast. Which of the following is indicated? A. Compression wrap for chest and breast trauma B. Mammogram C. Re-evaluation following complete resolution of obvious injuries D. Ultrasound

B. Mammogram Trauma to the anterior chest area may result in a palpable breast mass. An automobile accident with injury from contact with the seat belt, air bag, steering wheel, or dashboard is a common source of breast trauma. When a palpable mass results from chest trauma, it typically represents either a hematoma or area of secondary fat necrosis. Even when a mass is identified subsequent to direct trauma, the provider must remain suspicious for the possibility of malignancy that preexisted but was undetected before the accident.

Your 16-year-old patient reports that she gets her period sometimes every month and sometimes she skips a month. If menstrual periods are irregular, the term for this condition is: A. Menorrhagia B. Metrorrhagia C. Dysmenorrhea D. Amenorrhea

B. Metrorrhagia The menstrual history includes any episodes of amenorrhea, menorrhagia (excessive bleeding at the time of the menstrual cycle), metrorrhagia (bleeding at irregular noncyclic intervals), dysmenorrhea, and postmenopausal bleeding. Amenorrhea has many causes, including pregnancy; anorexia nervosa; excessive exercise; low body fat; and disorders or tumors of the hypothalamus, pituitary gland, ovary, uterus, and thyroid gland. Menorrhagia is most commonly caused by uterine fibroids, but hematologic disorders should be considered. Metrorrhagia can be caused by anovulation, intrauterine devices (IUDs), and ovarian and uterine tumors.

A 36-year-old female patient complains of foot pain when she wears high heels. There is no history of trauma or arthritis. On physical examination, there is tenderness at the space between the third and fourth metatarsal bones. Foot x-ray shows no evidence of stress fracture. This history is typical of: A. Plantar fasciitis B. Morton's neuroma C. Achilles tendonitis D. None of the above

B. Morton's neuroma Morton's neuroma, also called interdigital neuroma, is actually not neuroma but rather fibrous tissue thickening along digital nerves, typically in the space between the third and fourth intermetatarsals. They occur most frequently in women and may be associated with footwear, such as heels. Most common is localized pain that is increased with walking and decreased with rest and/or removal of shoes. Palpation often reproduces the sharp pain.

A 12-year-old boy is brought into the clinic by his mother. The teen boy is embarrassed because he has developed breast tissue and would like to know if there is any treatment. The physical examination reveals early stages of puberty, otherwise it is normal. This presentation is: A. Associated with a high rate of malignancy B. Most often due to altered hormonal levels in puberty C. Commonly indicative of pituitary tumor triggered by puberty D. Too dense for a mammogram and requires biopsy

B. Most often due to altered hormonal levels in puberty Gynecomastia most often occurs during infancy, puberty, and senescence. It is caused by an altered balance between estradiol and testosterone levels. Although it can be an indication of primary hypogonadism, hyperthyroidism, cirrhosis, or renal disease, the majority of the cases are specific to hormonal changes of puberty, are drug induced, or are idiopathic. With the presentation of breast enlargement in a male, malignancy must always be considered.

When carrying out a mental status exam on a non-English speaking patient, it is important to have a(n): A. Patient's family member, who is bilingual, interpret for the patient B. Objective interpreter ask the questions for the patient C. Friend of the patient interpret the questions for the patient D. Any of the above is acceptable

B. Objective interpreter ask the questions for the patient Ask the family or significant other whether the patient's behavior patterns have changed. If the patient does not speak or write English, have an interpreter available during the examination. It is important to have an objective interpreter ask the patient translated questions. The patient may not be truthful or forthcoming if he/she knows the interpreter. A friend or relative may slant the mental status exam questions or influence the patient regarding the answers. A good screening tool for use in the outpatient setting is the Mini-Mental Status Exam, described in detail in Chapter 20.

Your patient is a 46-year-old woman suffering from psychosis complains of milky discharge from both her breasts. Upon examination, there are no significant findings and the patient is not pregnant, breastfeeding, or in the postpartum period. You should: A. Order CT scan of head to rule out pituitary tumor B. Obtain a complete list of the patient's medications C. Have the patient keep a menstrual cycle diary D. Refer the patient for mammogram

B. Obtain a complete list of the patient's medications Galactorrhea is characterized by bilateral and milky discharge from multiple ducts in a woman who is neither pregnant nor lactating. Causes of galactorrhea include a variety of drugs as well as an elevated prolactin level associated with pituitary tumor or hyperthyroidism. The drugs associated with galactorrhea include antidepressants (amitriptyline, imipramine), psychoactives (haloperidol, thioridazine), hormones (estrogens, progestogens), antiepileptics (valproic acid), and antihypertensives (verapamil). This list is not exhaustive.

What is the most common cause of hip pain in older adults? A. Osteoporosis B. Osteoarthritis C. Trauma due to fall D. Trochanteric bursitis

B. Osteoarthritis There are many potential causes of hip pain. Among adults, the most common cause is OA with degenerative changes. In younger patients, the cause is often strain of the muscles or tendons. In comparison to other joints, the hip is often difficult to assess, in part because much of the joint and its periarticular structures lie deeper than those of other joints.

When performing a dipstick test on a patient's urine sample, a positive leukocyte esterase and nitrite is indicative of: A. Microscopic hematuria B. Urinary tract infection C. Calculi in the urine D. Possible bladder tumor

B. Urinary tract infection With urinary tract infection, urine dipstick will show positive leukocyte esterase, positive nitrite, and greater than 3 to 5 white blood cells per high-power field.

What kind of vertigo/loss of equilibrium also may include sensory disturbances? A) Vestibular neuronitis B) Meniere's disease C) Multiple sclerosis D) Acoustic neuroma

C) Multiple sclerosis

Your patient is a 63-year-old postmenopausal female with complaints of bloating, left lower quadrant abdominal pain, and feeling of abdominal fullness. On physical examination, the abdomen is non-tender with shifting dullness and there are enlarged left sided inguinal lymph nodes. Pelvic examination reveals slight adnexal tenderness. The clinician should have a high index of suspicion of: A. Endometriosis B. Ovarian cancer C. Uterine cancer D. Ovarian cyst

B. Ovarian cancer In the postmenopausal female, early symptoms of ovarian cancer are vague and include mild lower abdominal discomfort, feelings of fullness, bloating, distension, nausea, dyspepsia, constipation, and urinary frequency if the tumor is large. The CA-125 blood test by immunoassay can be used as a diagnostic tool. The characteristics typical of a malignant ovarian mass include solid, fixed, nodular, non-tender, and bilateral. The abdominal examination is essential looking for distension, changes in percussion, or ascites. As the cancer metastasizes, lymphadenopathy occurs, especially in the inguinal and supraclavicular areas.

A 75-year-old female patient complains of fatigue as well as pain and stiffness of the shoulders and neck. There is no history of trauma or exercised-induced pain. Medications include a beta blocker and ACE inhibitor. Medical history includes giant cell arteritis and Raynaud's syndrome. Physical examination is unremarkable. There is no swelling or erythema over the temporal arteries. There is no swelling, erythema, limitation in range of motion or point tenderness over the shoulder joints. The neck has normal range of motion and no tenderness or swelling. Which of the following disorders should be included in the list of possible diagnoses? A. Rheumatoid arthritis B. Polymyalgia rheumatic C. Drug-induced myalgia D. Fibromyalgia

B. Polymyalgia rheumatic Polymyalgia rheumatica is usually identified in adults aged 60 or older. The actual etiology of this condition is unknown. Giant cell arteritis occurs in about 15% of those with polymyalgia rheumatica, and the two conditions may be different expressions of the same etiology. The patient typically complains of sudden onset of widespread pain. Commonly affected sites include the neck, shoulders, and pelvis. Pain is accompanied by fatigue and stiffness. The stiffness is most profound in the morning. There is no actual muscle weakness. Unlike RA, there is no small joint inflammation and effusion.

Your 61-year-old female patient complains of dyspareunia. On pelvic examination, you note atrophic vaginitis. The clinician should explain that these are most likely: A. Signs of vaginismus B. Postmenopausal symptoms C. Signs of estrogen deficiency D. B and C

B. Postmenopausal symptoms There are no special examinations to be done for sexual dysfunction, although hormonal assessment may be helpful. A thorough gynecological examination is adequate to determine whether there is a physical cause for the problem. Decreases in libido and sexual response are not uncommon with aging, although it is an erroneous stereotype to assume that sexual desire or functioning automatically declines with age. In the aging client, lubricating agents may be helpful to deal with decreased or uncomfortable stimulation due to loss of lubrication. Decreased estrogen can cause atrophic vaginitis and thus decreased lubrication, leading to dyspareunia. Serum for free testosterone can be drawn if loss of libido is a problem in the postmenopausal woman.

Your 66-year-old patient complains of frequency of urination and hesitancy of the urine stream. On DRE, there is a hard, nodular, enlarged, non-tender prostate. The clinician should recognize these as symptoms of: A. Prostatitis B. Prostate cancer C. Urethritis D. Benign prostatic hyperplasia

B. Prostate cancer A prostate suspicious for malignancy will demonstrate nodular areas and/or overall hardness. Definitive diagnosis is made via prostate biopsy. Routine or urgent referral to a urologist is indicated, depending on the degree of PSA elevation and/or the degree to which it has risen since the previous value in conjunction with any suspicious findings on the rectal examination. A patient with prostate cancer often has no symptoms. Alternatively, an enlarged prostate can cause obstructive symptoms that include urinary urgency, frequency, hesitation in getting the stream started, decreased caliber and force of stream, and nocturnal frequency of urination that is bothersome.

A 43-year-old male patient complains of right-sided abdominal and pain in the back in the right costovertebral angle region, fever, chills, dysuria, and nausea. On physical examination, there is 102 degree fever, tachycardia, and right costovertebral angle tenderness to percussion. The most likely condition is: A. Lower urinary tract infection B. Pyelonephritis C. Nephrolithiasis D. Hydronephrosis

B. Pyelonephritis Pyelonephritis is a bacterial infection of the renal pelvis and parenchyma, typically caused by Escherichia coli ascending from the lower urinary tract. The patient will have bilateral or unilateral flank pain, fever, chills, nausea, and vomiting, and LUTS, such as dysuria, may also be present. The patient will appear ill on presentation, with fever and tachycardia commonly noted. Palpation and/or percussion over the infected side is painful. There may be accompanying abdominal discomfort or abdominal distension.

A 46-year-old female complains of fatigue, general malaise, and pain and swelling in her hands that has gradually worsened over the last few weeks. She reports that pain, stiffness, and swelling of her hands are most severe in the morning. On physical examination, you note swelling of the metacarpophalangeal joints bilaterally. These are common signs of: A. Osteoarthritis B. Rheumatoid arthritis C. Scleroderma D. Sarcoidosis

B. Rheumatoid arthritis RA typically affects the joints symmetrically. Symptoms may wax and wane, but the effects are cumulative and progressive. Although RA can affect any joint, it commonly affects the small joints of the hands and feet, and this is often helpful in diagnosis. There is often history of prolonged morning stiffness and fatigue. Affected joints are often tender, swollen with effusions, warm, and inflamed. The disease most commonly affects metacarpophalangeal and proximal interphalangeal joints.

You are an emergency room clinician that assisted with CPR on a 20-year-old trauma patient. The CPR was unsuccessful, the patient expired, and you need to explain this to the family in the waiting room. Upon telling the mother about the death of her son, she becomes dizzy and faints. What is the most likely cause for the woman fainting? A. Cardiogenic shock B. Vasovagal response C. Syncope due to hypoxia D. Dizziness and vertigo

B. Vasovagal response Neurocardiogenic syncope, also called vasovagal syncope, is a common cause of dizziness and fainting. It is due to a sudden decrease in blood pressure and heart rate after prolonged standing, with stress, or from dehydration. It is a result of sympathetic sensitivity, causing a reflexive response that suddenly causes bradycardia and venous dilation. Hypotension and dizziness result.

Ménière's' disease presents with the following triad of symptoms: A. Vertigo, nystagmus, hearing loss B. Vertigo, tinnitus, hearing loss C. Vertigo, syncope, hearing loss D. None of the above

B. Vertigo, tinnitus, hearing loss The exact cause of Ménière's disease is unknown. However, the symptoms are associated with increased fluid and pressure in the labyrinth. Ménière's disease commonly involves a triad of symptoms: severe vertigo, tinnitus, and hearing loss. The vertigo is transient but recurrent. The tinnitus and hearing loss may also be intermittent and/or recurrent but often become worse over time. A sensation of ear fullness may precede an episode. During the episode, vertigo is often debilitating and is associated with nausea and vomiting. The tinnitus and hearing loss are usually unilateral.

An 88-year-old female is accompanied by her daughter to the emergency department. The daughter explains that her mother was having her hair washed at the beauty salon, then complained of dizziness and fainted. The 88-year-old patient has a history of hypertension for which she takes a beta blocker. Otherwise she was in her usual state of health until this incident. On examination, she has no neurological deficits. Which of the following conditions should be considered? A. Benign positional vertigo B. Wertebrobasilar insufficiency C. Labyrinthitis D. Dehydration

B. Wertebrobasilar insufficiency Vertebrobasilar insufficiency is seen mostly in the elderly and is exacerbated by extension of the neck or changes in head position. There is temporary interruption of circulation in the posterior brain due to vertebral artery occlusion. Benign paroxysmal positional vertigo, or cupulolithiasis, is the most common vestibular disorder, resulting from otolithic crystals/particles detaching from the utricle membrane and migrating to the semicircular canal. It can occur spontaneously with motion or position change or as a result of vascular or labyrinth trauma. Characterized by sudden-onset dizziness lasting less than 30 seconds and following a head position change, cupulolithiasis may be accompanied by nystagmus. It usually subsides but may recur at any time. Labyrinthitis is caused by the invasion of the ear by bacteria or a virus. The bacterial version is more serious because it may lead to meningitis. Prompt treatment with antibiotics is necessary. Labyrinthitis is characterized by severe vertigo, nystagmus, and hearing loss. Suppurative labyrinthitis may be secondary to bacterial otitis media or other bacterial infection. Serous labyrinthitis can be secondary to a variety of viral illnesses, including measles, mumps, chickenpox, influenza, mononucleosis, and adenovirus.

A 23-year-old female presents with ankle pain due to a fall while walking on ice 1 hour ago. You watch her limp as she walks into the emergency room. On physical examination, the ankle is erythematous, swollen, and tender to touch. The patient cannot stand on the affected ankle. Range of motion is severely limited on inversion of the ankle. On palpation, there is no pain with pressure on the medial or lateral malleolus. Should this patient be sent for ankle x-rays according to the Ottawa ankle rules? A. No, the ankle shows no tenderness over the medial or lateral malleolus B. Yes, the patient is unable to bear weight on the ankle C. Yes, ankle range of motion is limited D. No, range of motion of the ankle is not limited in all planes

B. Yes, the patient is unable to bear weight on the ankle The following Ottawa rules recommend x-ray of the ankle if either one of the following conditions exist: - Inability to bear weight for four steps (both immediately and in emergency department) - Bone tenderness at posterior edge or tip of either malleolus

A CBC test can be used to diagnose which of the following? A) Hip fracture B) Sciatica C) Bursitis D) Osteoarthritis

C) Bursitis

Which of the following disorders has a strong genetic component and causes loss of spinal mobility and progressive erosion of the sacroiliac joint? A. Syringomyelia B. Spinal stenosis C. Ankylosing spondylitis D. None of the above

C. Ankylosing spondylitis Ankylosing spondylitis is one of the spondyloarthropathies, which have genetic predispositions and are inflammatory disorders. Early symptoms include LBP and stiffness, which gradually become persistent and increase in severity. There is loss of spine mobility, and posture gradually changes with flexion of the neck, increased kyphosis of the thoracic region, and loss of the lumbar curve. The gene HLA-B27 is present in most patients. Radiographs show abnormality of the sacroiliac joint with progressive erosion. A CT scan is useful to identify sacroiliitis.

Your patient has a rubbery, firm, mobile breast mass. In order to completely exclude the possibility of cancer, the triple test is necessary, which includes: A. CT scan B. MRI C. Biopsy D. All of the above

C. Biopsy Palpation and mammography, alone or together, are inadequate to definitively identify the cause of a breast mass and to rule out malignancy. The ―triple test‖ recommended for evaluation of a breast mass involves clinical examination, either ultrasound or mammogram, and aspiration and/or biopsy. The determination of whether an ultrasound or mammography is recommended is based on age and other situations.

Which of the following microorganisms causes Lyme disease? A. Clostridia B. Shigella C. Borrelia D. Epstein-Barr virus

C. Borrelia Lyme disease is caused by the bacterium Borrelia burgdorferi and is transmitted by a bite from a deer tick. Whereas the incubation period ranges from 3 to 30 days, the onset of symptoms typically appears in 7 to 14 days. Although the disorder can be asymptomatic, the patient generally develops migratory polyarthralgia, myalgia, and neurological findings, including meningitis and/or neuropathy. An early finding is a solitary target lesion that may be followed by multiple lesions.

Your patient with a cervical herniated disc presents with a weakened biceps reflex. The biceps reflex is a test of the following spinal nerves: A. C3 to C4 B. C4 to C5 C. C5 to C6 D. C7 to T1

C. C5 to C6 Box 15.3 Deep Tendon Reflexes: - Biceps: C5, C6 - Brachioradialis: C5, C6 - Triceps: C6, C7 - Patellar: L3, L4 - Achilles: S1

Which of the following findings on the bimanual exam of a female patient requires further investigation? A. Retroverted uterus B. Varicose veins in vaginal wall C. Cervical motion tenderness D. Anteflexed uterus

C. Cervical motion tenderness The uterus should be pear-shaped, smooth, mobile, non-tender, and firm but not hard. Malignancies may occur, causing the uterus to be hard and fixed. Tenderness with movement of the cervix and uterus occurs in PID, termed cervical motion tenderness. Variations in the position of the uterus are not considered abnormal, but significant variations may be related to back pain, especially during menses or childbirth, and occasionally to infertility. Common uterine positions are retroverted, retroflexed, anteverted, and anteflexed.

A 15-year-old female patient presents with complaints of constant thirst, urination up to 20 times a day, and recent unintentional weight loss of 10 pounds over the last month. It is important for the clinician to recognize these symptoms as those of: A. Renal cancer B. Bladder cancer C. Diabetes mellitus D. Interstitial cystitis

C. Diabetes mellitus Changes to the fluid and electrolyte balance of the body can directly affect urinary output, such as in the osmotic diuresis, seen with poorly controlled or undiagnosed diabetes mellitus. Other examples include diabetes insipidus, metabolic acidosis and alkalosis, renal insufficiency, and congestive heart failure.

Your 35-year-old female patient complains of severe cramping with monthly menstrual flow. Painful menstrual periods are termed: A. Menorrhagia B. Metrorrhagia C. Dysmenorrhea D. Premenstrual syndrome

C. Dysmenorrhea Primary dysmenorrheal, which typically presents as severe cramping during menstruation, is common and generally does not indicate pathology, particularly in the young population. It is most severe in the first few days of the menstrual cycle.

Your patient is a 43-year-old female golfer who complains of arm pain. On physical examination, there is point tenderness on the elbow and pain when the patient is asked to flex the wrist against the clinician's resistance. These are typical signs of: A. Carpal tunnel syndrome B. Osteoarthritis of the wrist C. Epicondylitis D. Cervical osteoarthritis

C. Epicondylitis Epicondylitis involves inflammation of the tendon/tendon insertion of the forearms. This tendinitis results in either lateral elbow pain associated with overuse of the wrist extensors (tennis elbow) or in medial elbow pain associated with overuse of involving wrist flexion and rotation (golfer's elbow). Point tenderness is noted at the medial or lateral epicondyle. The onset and severity of pain is usually gradual and progressive but may have relatively acute onset following an activity involving significant repetitive use. The pain may be referred to the forearm and is increased by the offending motion (wrist flexion, extension, or rotation). Pain is usually greater when the motion is made against resistance.

In dementia, which of the following cognitive functions is most commonly lost first? A. Recognizing persons B. Knowing the place C. Estimating the time D. Long-term memory

C. Estimating the time When evaluating a patient's mental status, assess orientation by asking the patient to recite his or her full name, current location/place (clinic, hospital, home, etc.), and the date (day of the week, month, or year). Knowledge of time is generally impaired first, followed by place. The inability to recite or recognize one's name implies a significant deficit in mental status.

A 39-year-old female patient comes to the clinic for a pelvic exam and Pap smear. She reports that her partner is infected with condyloma acuminata. In her history, she always has her partner use a condom. The external genitalia, vagina, and cervix appear normal on pelvic exam, but the Pap smear shows dysplasia. The patient is worried that she could be infected with HPV. The clinician should: A. Assure the patient that safe sexual practices protect against HPV infection B. Explain that there is no condyloma present, so HPV infection is improbable C. Explain that HPV infection can be transmitted in the presence of barrier contraception D. Teach the patient about hysterectomy since she has cervical cancer

C. Explain that HPV infection can be transmitted in the presence of barrier contraception There is an increased incidence of cervical cancer in women infected with HPV, with neoplasias being associated with types 16 and 18. Visible manifestations of HPV occur in less than a dozen genotypes, estimated at only 1% of those infected. Of those who develop condylomas, the presentation is variable. Lesions are often very small, even microscopic, and the rugae of the vaginal mucosa may mask the lesion. If dysplasia is seen on the Pap smear, biopsies should be taken to rule out intraepithelial neoplasia because more than 90% of the cervical neoplasias are caused by HPV. Unfortunately, barrier contraceptives offer only limited protection against the spread of HPV infection.

A 33-year-old female reports general malaise, fatigue, stiffness, and pain in multiple joints of the body. There is no history of systemic disease and no history of trauma. On physical examination, the patient has no swelling or decreased range of motion in any of the joints. She indicates specific points on the neck and shoulders that are particularly affected. She complains of tenderness upon palpation of the neck, both shoulders, hips, and medial regions of the knees. The clinician should include the following disorder in the list of potential diagnoses: A. Osteoarthritis B. Rheumatoid arthritis C. Fibromyalgia D. Polymyalgia rheumatica

C. Fibromyalgia In fibromyalgia, the most common symptoms are generalized pain, stiffness, and decreased ROM, with multiple-point tenderness. The diagnostic criteria currently rest on a patient reporting point tenderness in at least 11 of 18 specified sites (Fig. 14.1) in addition to the presence of widespread pain for at least 3 months. The most common tender sites are in the neck, shoulders, spine, and hips. Other common symptoms include morning stiffness, anxiety, depression, sleep disturbances, "brain fog," and irritable bowel syndrome.

An intravenous pyelogram should not be performed if serum creatinine is: A. Less than 1.5 B. Greater than 1.0 C. Greater than 1.6 D. Less than 1.0

C. Greater than 1.6 IVP (also known as intravenous urography or excretory urography) is a study for noninvasive evaluation of the renal pelvis and ureter, with moderate cost and ease of administration. It demonstrates a wide variety of upper tract lesions and is well tolerated by most patients with a serum creatinine of 1.6 or less.

A 17-year-old female patient complains of amenorrhea for the last 8 months, weight gain, excessive hair growth on the arms and chest, and development of acne. Pelvic examination is normal. The clinician should recognize these are signs of: A. Ovarian cancer B. Endometriosis C. Hormonal imbalance D. Stress-induced anovulation

C. Hormonal imbalance Anovulation may present with amenorrhea but also may present with dysfunctional uterine bleeding, polymenorrhea, or menorrhagia. The symptoms vary with the cause. Overweight may be seen with several of the causes, including hypothyroidism, polycystic ovarian syndrome, and pituitary and adrenal dysfunction. Underweight is seen in anorexia nervosa, excessive exercise, hyperthyroidism, or stress-induced anovulation. Hirsutism, acne, and other skin changes can be seen with imbalances in LH, FSH, and androgens, as seen in polycystic ovary disease. Delayed puberty or regression of sexual characteristics is seen in hypopituitarism; galactorrhea can be the presenting symptom in pituitary tumors.

A 33-year-old female patient visits the clinic multiple times over the last year complaining of dysuria, frequency, urgency, suprapubic pain, and dyspareunia. Her physical examination, including a pelvic exam, is normal. Urinalysis and urine culture is repeatedly negative. Her intravenous pyelogram is also normal. The clinician should recognize that this presentation is consistent with symptoms of: A. Asymptomatic bacteriuria B. Upper urinary tract infection C. Interstitial cystitis D. Polycystic kidneys

C. Interstitial cystitis Patients with IC suffer from chronic symptoms that include a combination of dysuria, suprapubic pain, chronic pelvic pain, dyspareunia, and negative urine cultures. Patients may become debilitated by this disease; they may be making up to 40 trips to the bathroom in 24 hours. IC is also marked by periods of remission and flare-up throughout a patient's lifetime. Typical age at onset varies from 30 to 70, and most patients visit an average of five physicians and wait 4 years before the correct diagnosis is made.

The definitive diagnosis of endometriosis is made with: A. Hysterosalpingoscopy B. Pelvic CT scan C. Laparoscopy D. Pelvic MRI

C. Laparoscopy The history and physical examination generally lead you to suspect endometriosis, but the definitive diagnosis is made through laparoscopic surgery and biopsy. The lesions seen in endometriosis, termed endometrial implants, appear as dark red to dark brown lesions that give the appearance of a powder burn from a gunshot. They appear on the peritoneal, bladder, uterine, and ovarian surfaces.

In order to perform interventions or prescribe medication for the female patient, it is most important to obtain a complete health history. Which of the following is the most important question to ask female patients? A. Age at menarche B. Age at menopause C. Last menstrual period D. Number of pregnancies

C. Last menstrual period Last menstrual period is one of the most important questions to ask, particularly when prescribing medications, because many are contraindicated in pregnancy. If menstrual cycles are not regular, pregnancy should be ruled out first, and then other diagnoses can be considered.

A 62-year-old female presents with a singular, hard, 1 cm, non-tender, non-mobile mass in the right breast. There are no nipple or skin changes, however, you palpate an enlarged right-sided supraclavicular lymph node. The clinician should recognize these are signs of: A. Fibroadenoma B. Breast cyst C. Malignancy D. Paget's disease

C. Malignancy The typical malignant breast mass is solitary, non-tender, hard, immobile or fixed, and poorly defined. It may be accompanied by nipple erosion or other inflammatory skin changes, as seen in Paget's disease; nipple discharge; skin thickening or dimpling; retraction; and palpable axillary nodes. Although most malignant masses are painless, associated discomfort does not exclude the potential for breast cancer.

A female patient presents to the clinic with complaints of a severe, throbbing, unilateral headache. She complains of seeing flashes of light prior to the headache. She complains of sound and light sensitivity as well as nausea. The clinician should recognize these as symptoms of: A. Epilepsy with aura B. Cluster headache C. Migraine headache D. Normal pressure hydrocephalus

C. Migraine headache Typical migraine pain begins unilaterally but may become generalized and may lateralize to the opposite side and/or radiate to the face or neck. The pain ranges from a dull ache to a throbbing or pulsatile pain. The pain is often severe and/or incapacitating and is often aggravated by movement, light, and noise. Accompanying symptoms may include nausea, vomiting, photophobia, phonophobia, osmophobia, dizziness, chills, and/or ataxia. There may be tenderness to palpation of the temporal arteries. Auras, if experienced, may include blurred vision and scotoma and/or other prodromal symptoms, such as anorexia, irritability, restlessness, or paresthesias lasting from 30 minutes to 3 hours before the onset of migraine pain.

Your 24-year-old female patient complains of tenderness at the entrance of the vagina on the right side. She cannot sit comfortably because of the soreness in the perineal area. On pelvic exam, there is of a tender, inflamed, fluctuant cyst at the introitus on the right side. These signs and symptoms are indicative of: A. Possible malignancy B. Bartholin cyst C. Nabothian cyst D. Candidiasis

C. Nabothian cyst The two Bartholin's glands are mucus-secreting glands, one located in each lateral wall of the vagina at the introitus. They can become abscessed in acute or chronic inflammatory processes, usually as a result of STD infection or from other bacteria that are prevalent in the vaginal and perineal area but also by thickened mucus or congenital narrowing of the duct. In postmenopausal women, a malignant cause should be considered in the differential diagnosis. Acutely, Bartholin's abscesses are extremely painful, in part due to their anatomical location, which makes sitting difficult. The area becomes swollen with the presence of a tender, inflamed, fluctuant cyst that is easily visualized.

Which type of pelvic disorder has the highest rate of mortality in the United States? A. Uterine cancer B. Cervical cancer C. Ovarian cancer D. Pelvic inflammatory disease

C. Ovarian cancer Ovarian cancer has the highest mortality of the gynecological malignancies because as much as 85% of cases have metastases outside the ovaries at the time of diagnosis, and the cancer is usually widespread before the patient has signs or symptoms. It occurs most often in postmenopausal women, in women with a positive family history, and in women with BRCA1 and BRCA2 gene mutations. Use of oral contraceptives may decrease a woman's risk of developing ovarian cancer.

Your patient is a 64-year-old female who presents with complaints of a red, scaly rash on her nipple and areola of one breast. It has been present for a few months. There is no mass or nipple discharge. You should be suspicious of: A. Contact dermatitis B. Mastitis C. Paget's disease D. Eczema

C. Paget's disease In Paget's disease of the breast, the patient may describe the persistence of skin changes for several months. The typical presentation involves skin changes of the nipple and/or areola, with the nipple being involved first. The condition does not always involve a palpable mass or nodule. The skin changes of the nipple and areola range from scaling redness to various degrees of ulceration.

A 65-year-old woman is accompanied by her daughter for a physical examination. She has mild heart failure and takes digitalis and an ACE inhibitor. As you examine the patient, you note flat affect, hand tremor, and slowed movements. The tremor is worsened at rest. There are no neurologic deficits. Hand grip, sensation of face and extremities, and lower extremity muscle strength are within normal limits and bilaterally equal. DTRs are equal bilaterally. CN II to XII are intact. The mental status exam is normal. These are key signs of: A. Chiari malformation B. Normal pressure hydrocephalus C. Parkinson's disease D. Drug toxicity

C. Parkinson's disease Parkinson's disease occurs with approximately equal sex distribution, and usually begins between 45 and 65 years of age. Unilateral pill-rolling tremor at rest is usually the first symptom. The tremor is maximal at rest but absent during sleep and can be differentiated from essential tremor, which is absent at rest and worsens with voluntary movement. There is a flattened affect and blank stare. There is bradykinesia of gross and fine motor movement, speech volume, swallowing, and blinking. There is generally no muscle weakness, and deep tendon reflexes are normal. Although Alzheimer's disease can manifest with rigidity, bradykinesia, and gait disorders, no resting tremor is seen with Alzheimer's.

Your 45-year-old female patient complains of irregular menstrual cycles, severe pain with menses and heavy bleeding every month for the last year. On examination, you palpate an enlarged uterus. Pregnancy test is negative. A recommended diagnostic test for this patient is: A. Abdominal CT scan B. Kidney, ureter, bladder x-ray C. Pelvic ultrasound D. Abdominal MRI

C. Pelvic ultrasound Uterine leiomyomas, more commonly known as uterine fibroids, are benign growths consisting mostly of smooth muscle. The etiology is unknown, but their growth is hormone dependent; therefore, they are seen in approximately 25% of women during their reproductive years. Heavy menstrual bleeding (menorrhagia) and irregular bleeding (metrorrhagia) are the most common presenting symptoms, although a large percentage of patients are asymptomatic. Other symptoms include heaviness or fullness in the lower abdomen, pelvic pain, backache, dysmenorrhea, and urinary complaints. Most leiomyomas can be palpated on bimanual examination, and some larger fibroids can be palpated through the abdomen. The uterus may feel enlarged, irregular, or nodular. A pregnancy test should be performed to rule that out as a cause of the symptoms. A CBC is needed in cases of heavy bleeding to determine whether anemia or platelet disorder is present. Pelvic ultrasound or MRI should be performed if symptoms of leiomyoma are present or for any palpable pelvic mass detected on physical examination.

A 60-year-old female complains of several episodes of dizziness and nausea that started this morning. She denies trauma, falling, or loss of consciousness. She has a 5-year history of hypertension and takes a beta blocker daily. On physical examination, there are no neurologic deficits or any abnormal findings and ECG is normal. Which of the following tests should be performed? A. Test the patient for Kernig's sign B. Test the patient for Brudzinski sign C. Perform the Hallpike maneuver D. Assess for Babinski's sign

C. Perform the Hallpike maneuver Characterized by sudden-onset dizziness lasting less than 30 seconds and following a head position change, benign positional vertigo (cupulolithiasis) may be accompanied by nystagmus. It usually subsides but may recur at any time. In addition to the history, a provocative test for positional nystagmus can be performed (Hallpike maneuver), although it is not always positive. The provocative test involves moving the patient quickly from a sitting position to a lying position with the head turned to the side and the head dependent over the side of the examination table. After a few seconds, vertigo and nystagmus occur. This response fatigues with immediate repetition of the test.

Which of the following is a sign of glenohumeral instability? A. Positive Lachman test B. Negative Spurling's sign C. Positive apprehension test D. Negative Lhermitte sign

C. Positive apprehension test Unlike the other conditions affecting the shoulder, glenohumeral instability is most common in young patients who are physically active. The instability can result in displacement of the humeral head in various directions. The patient will experience sudden onset of pain and be unwilling to move the arm. The displacement may follow an acute injury/trauma or may be associated with specific movements or overuse. A positive apprehension test suggests glenohumeral instability. This can be somewhat validated by performing the relocation test, in which the apprehension test is immediately followed by placing mild anterior pressure on the arm paired with external rotation.

On DRE, you note that a 45-year-old patient has a firm, smooth, non-tender but asymmetrically shaped prostate. The patient has no symptoms and has a normal urinalysis. The patient's PSA is within normal limits for the patient's age. The clinician should: A. Refer the patient for transrectal ultrasound guided prostate biopsy B. Obtain an abdominal x-ray of kidneys, ureter, and bladder C. Recognize this as a normal finding that requires periodic follow-up D. Obtain urine culture and sensitivity for prostatitis

C. Recognize this as a normal finding that requires periodic follow-up An asymmetric prostate is typically asymptomatic and not necessarily diagnostic of prostate cancer; asymmetry can be a normal finding on DRE but should be followed periodically to monitor for changes. Age-specific reference ranges for PSA (see Table 11.4) should be used as a guide when there is no previous PSA for comparison. A prostatic nodule found on DRE necessitates a referral to a urologist or radiologist for transrectal ultrasound-guided prostate biopsy and may well be the first indication of the presence of a cancer.

A 23-year-old female presents to the emergency room with fever, pelvic pain, vomiting, and dysuria. She reports having had an abortion yesterday in an urban birth control clinic. On physical examination, there is a bloody vaginal discharge, tenderness of the left lower quadrant of the abdomen, and cervical tenderness on pelvic examination. The following is a likely diagnosis: A. Neisseria gonorrhea infection B. Chlamydia trachomatis infection C. Staphylococcal endometritis D. Fulminant candidiasis

C. Staphylococcal endometritis PID is defined as an infection of the uterus, fallopian tubes, and adjacent pelvic structures. It is often secondary to an STD or other infection of the lower reproductive tract that migrates upward into the uterus and tubes. N. gonorrhea and C. trachomatis are two of the commonly offending organisms. Pelvic infections can also occur postsurgery, postpartum, or postabortion but are generally caused from other organisms, such as staphylococcus or streptococcus. Abdominal pain, mucopurulent cervical discharge, and often fever are the more common presenting symptoms. Rebound tenderness indicates peritoneal irritation. Dysuria, nausea, and vomiting may also be present. The abdominal pain is midline and often accompanied by right and left lower quadrant pain, particularly when accompanied by salpingitis. During the pelvic examination, there is pain with cervical motion and with palpation of the uterus and ovaries.

An 81-year-old patient with heart failure comes into the emergency room accompanied by his daughter. The daughter reports that her father "banged his head" as he was getting out of the car 4 days ago. He did not complain of headache pain, so she did not obtain medical advice. She reports that she noticed a mild facial droop, slurring of speech, and gait disturbance in her father today. She reports that her father takes Coumadin, digoxin, and an ACE inhibitor. On physical examination, there is decreased right hand grip 2/5 strength and decreased right quadriceps strength 2/ 5 compared to the left side 5/5. CT scan shows an intracranial bleed. Which of the following is the most probable diagnosis? A. Subarachnoid hemorrhage B. Epidural hematoma C. Subdural hematoma D. Intracerebral hemorrhage

C. Subdural hematoma Subdural hematomas can be either acute or chronic. Acute subdural hematomas are usually associated with an acute head injury and can cause a range of symptoms, including headache and loss of consciousness. A chronic subdural hematoma in the elderly population may enlarge significantly before the patient begins to notice head pain. The headache associated with subdural hematoma is generally dull and aching in nature and may be transient. The history often includes a blow to the head, fall, or other injury, which preceded the pain. The physical findings vary depending on the severity of the trauma but may include progressive neurological deterioration, which may advance to include coma. The elderly patient with head trauma and anticoagulants should raise suspicion of subdural hematoma.

A 34-year-old female presents with fever, general malaise, fatigue, arthralgias and rash for the last 2 weeks. On physical examination, you note facial erythema across the nose and cheeks. Serum diagnostic tests reveal positive antinuclear antibodies, anti-DNA antibodies, elevated C-reactive protein and erythrocyte sedimentation rate. The clinician should include the following disorder in the list of potential problems: A. Fibromyalgia B. Sarcoidosis C. Systemic lupus erythematosus D. Rheumatoid arthritis

C. Systemic lupus erythematosus SLE has many potential symptoms. The classic findings include a malar rash. Patients often have arthralgias, myalgias, fever, fatigue, Raynaud's syndrome, and neuropathy. SLE effects depend on the organs involved and diagnosis can be difficult. A positive ANA occurs at some point in the condition in the majority of patients but is neither consistent nor specific for SLE. Positive anti- DNA and lupus erythematosus prep are also common to SLE.

Which of the following is true of palpation of the prostate gland? A) It is not done. B) It is part of a patient's self-examination routine. C) It is usually done externally. D) It is usually done during a rectal examination.

D) It is usually done during a rectal examination.

Which of the following is true of the kidneys? A) The right kidney usually lies slightly higher than the left. B) The renal cortex is the outer portion of the kidney. C) The renal medulla contains nephrons. D) Nephrons are considered the functioning units of the kidney.

D) Nephrons are considered the functioning units of the kidney.

A 56-year-old woman brings her 78-year-old father to the emergency room. The patient complains that sometimes he can't think straight. His daughter reports that her father has been "tripping over his own feet" and has become incontinent of urine in the last few days. The patient's medications include an ACE inhibitor and beta blocker. On physical examination, vital signs are within normal limits, heart and lungs show no abnormalities, cranial nerves are intact, and sensation and muscle strength are normal. A mental status exam is normal. Imbalanced gait and a positive Romberg test are apparent. Which of the following are possible disorders? A. Cerebellar dysfunction B. Normal pressure hydrocephalus C. Early dementia D. A & B

D. A & B Normal pressure hydrocephalus is not fully understood. It is seen primarily in persons over 60 years of age and involves enlargement of the ventricles, often without increased CSF pressure; intraventricular pressures may be high or normal. One of the theorized causes includes intermittent pressure increases. It is slightly more common in men than in women. The patient often first notices some degree of gait disorder, followed by the onset of a "clouding" of thought processes, which gradually progress. The typical picture is a patient who has a triad of gait disturbance, altered thought processes, and urinary incontinence. Strength and sensation are usually within normal limits. However, focal neurological findings are present and include increased deep tendon reflexes, the inability to tandem walk, positive Babinski, and/or positive Romberg.

To confirm the diagnosis of amenorrhea due to menopause, blood should be drawn for hormones. Which are the changes that occur with menopause? A. FSH rises B. LH decreases C. LH rises D. A & C

D. A & C In menopause, FSH rises first and then LH rises, both greater than 100 mU/mL. A fall in estradiol is the last hormonal change that occurs with the decline of ovarian function. An estradiol level of less than 30 pg/mL indicates loss of ovarian function.

A 20-year-old female complains of thin vaginal discharge and pruritus. On pelvic examination, there are "strawberry spots" on the vaginal walls. With this infection, it is important to: A. Treat the patient's partner B. Perform STD and HIV testing C. Look for characteristic clue cells D. A and B

D. A and B The presenting complaints with trichomoniasis are discharge and itching. It can be differentiated from yeast by the discharge, which is thin and frothy rather than the thick, curdlike discharge of yeast. It can also be differentiated from G. vaginalis by the presence of vulvar itching and inflammation with trichomoniasis but no complaint of odor, as there is with G. vaginalis. Inflammation with petechiae of the vaginal walls, known as ―strawberry spots,‖ is diagnostic of T. vaginalis. Male partners are usually asymptomatic but harbor the organism, and they must be treated along with the patient; intercourse should be avoided or condoms used until treatment is completed.

Your patient is 40-year-old baseball player who needs his yearly physical exam. He reports a 5- year history of chronic lower back pain due to spinal stenosis. Which of the following findings indicate spinal stenosis? A. Lumbar x-ray demonstrates vertebral osteophytes B. Positive straight leg raising sign upon physical exam C. Lumbar MRI shows decreased intervertebral space D. A and C

D. A and C Caused by progressive degenerative spine changes, spinal stenosis is most common at middle age or later. Spinal stenosis pain is usually worse during the day. It is aggravated by standing and relieved by rest. The pain varies from severe to mild. OA signs may be present. Radiological findings may indicate extensive vertebral osteophytes and degenerative disk disease. An MRI or CT scan can be helpful if initial x-rays are inconclusive.

A 43-year-old female was in a bicycling accident and complains of severe pain of the right foot. The patient limps into the emergency room. On physical examination, there is no point tenderness over the medial or lateral ankle malleolus. There is no foot tenderness except at the base of the fifth metatarsal bone. According to the Ottawa foot rules, should an x-ray of the feet be ordered? A. Yes, there is tenderness over the fifth metatarsal B. No, there is not tenderness over the navicular bone C. Yes, the patient cannot bear weight on the foot D. A and C

D. A and C Foot Rule: Order film if one of the following is met: - Inability to bear weight for four steps (both immediately and in emergency department) - Bone tenderness at navicular or base of fifth metatarsal - Sensitivity = 100% - Specificity = 79%

A 56-year-old male presents with complaints of right-sided neck pain with radiation down the right arm into the right index finger and thumb. He reports a recent fall from a scaffold while painting on the job. On physical examination, you note a 0/4 biceps reflex on the right compared to 2/4 on the left and 1/5 grip strength of the right hand compared to 5/5 of the left hand. Which of the following is a test that can be performed to assist in the diagnosis of cervical disk herniation? A. Spurling's sign B. Apley's sign C. Lhermitte's sign D. A and C

D. A and C In cervical spine trauma, cervical disk herniation can occur. A positive Spurling's sign is noted if this maneuver reproduces neck and radicular pain, suggesting a herniated disk. Spurling's sign is tested for by lightly pressing downward on the top of the patient's head while tilting back and toward the side of pain. Lhermitte's sign may support suspicion of a herniated disk. This test is conducted by having the patient flex the neck in a chin-to-chest motion and is positive if an electric shock-like sensation down the spine results.

A 34-year-old baseball pitcher complains of pain in the left shoulder, particularly with raising the left arm when attempting to pitch. Which of the following is a test used to diagnose rotator cuff injury? A. Apley's test B. Trousseau test C. Hawkin's test D. A and C

D. A and C The rotator cuff consists of the supraspinatus, infraspinatus, subscapularis, and teres minor. Injury to the rotator cuff is typically due to chronic impingement with degenerative changes over time. The patient typically complains of anterior and lateral shoulder pain that increases with arm elevation and reaching overhead. The pain is usually progressive and may be associated with repetitive activities. Pain at night may cause sleep disturbance. ROM is typically preserved. Apley's (see Fig. 14.2) and Hawkins' (see Fig. 14.3) tests may reproduce the pain, depending on the component of the rotator cuff involved. There may be point or diffuse tenderness to the shoulder area. Crepitus and/or arm weakness suggest an acute tear.

To assess muscle tone, the clinician should: A. Palpate the patient's muscle as the muscle is passively stretched B. Examine how the patient performs active range of motion C. Palpate the muscles comparing side to side D. A and C

D. A and C To assess muscle tone, passively stretch the muscle, ask the patient to relax, and then palpate the muscle, comparing side to side. Alternatively, assessment of muscle tone can be combined with a determination of the patient's resistance to passive movement. Tense patients, those with increased muscle tone, will have increased resistance to passive movements. Flaccid or hypotonic muscles do not have any palpable tension. A spastic muscle has increased resistance, which may vary as the limb is moved, as in ―cogwheeling,‖ such as that found in patients with parkinsonism. Resistance with both flexion and extension is called lead-pipe rigidity, as is sometimes seen in parkinsonism.

When examining the carotid arteries, the clinician should: A. Ask the patient to hold his/her breath while auscultating the carotid arteries B. Use the bell of the stethoscope to listen over the carotid arteries C. Palpate one carotid artery at a time D. All of the above

D. All of the above Auscultation of the carotid arteries is an important portion of the neurological examination, particularly for elderly patients. The patient should be asked to hold his or her breath during auscultation. The bell of the stethoscope is used to auscultate for bruits. One carotid artery at a time is palpated. A bruit in the carotid artery may be an indicator of potential stroke or carotid artery stenosis and should be followed by further tests, such as carotid duplex and carotid ultrasound.

An important sign that indicates nerve compression at the cauda equina section of the spinal cord is: A. Urinary incontinence B. Urinary retention C. Bladder spasms D. All of the above

D. All of the above Neurogenic (or neuropathic) bladder is the failure of the bladder to store or to empty. There may be spontaneous and uncoordinated contractions of the bladder when it is filling, or the bladder and sphincter may not work in concert, preventing the bladder from effectively emptying. Risk factors include spinal cord injury, trauma to the central nervous system, diabetes mellitus, spina bifida, multiple sclerosis, spinal disk disease, and pelvic surgery, among many others. A variety of urinary complaints may occur, including incontinence, dribbling, and retention as well as disorders of bladder sensation.

A 20-year-old male construction worker is experiencing new onset of knee pain. He complains of right knee pain when kneeling, squatting, or walking up and down stairs. On physical examination, there is swelling and crepitus of the right knee and obvious pain with resisted range of motion of the knee. He is unable to squat due to pain. Which of the following disorders should be considered in the differential diagnosis? A. Joint infection B. Chondromalacia patella C. Prepatellar bursitis D. All of the above

D. All of the above Prepatellar bursitis is also called housemaid's knee, which is common to persons whose occupation requires extended periods of kneeling, such as plumbers and carpet layers. This bursitis can also be caused by an infection. The patient complains of pain in the area inferior to and over the patella, and there is swelling and inflammation of the bursa. Chondromalacia patella is seen in young active persons of either gender. The condition is also commonly called patella-femoral syndrome and runner's knee. The pain involves the anterior knee, often develops gradually, and is moderate in intensity. Pain can be reproduced by pressing the patella against the femoral condyles, and there is tenderness around the patella. Other maneuvers that reproduce the pain include applying pressure against the patella as the patient extends the lower leg, flexing the quadriceps, and moving the patella from side to side. Crepitus and effusion are often present. - Decision Rule in Chondromalacia Patella: Two or more of the following symptoms (pain reported on muscle contraction, squatting, or kneeling) is associated with 60% sensitivity and 85% specificity for patellofemoral pain syndrome. Of these, the one maneuver with greatest diagnostic strength is pain on resisted contraction (Cook et al., 2010).

A 34-year-old female patient complains of lack of conception after unprotected sexual intercourse for 1 year. The patient has been keeping track of basal body temperature to plot ovulation. Pelvic examination is normal. The clinician should ask the patient if which of the following laboratory test(s) have been done: A. TSH B. FSH C. LH D. All of the above

D. All of the above The work-up for infertility is complex and will need to be referred to a gynecologist specializing in infertility. The causes are numerous but generally fall into the categories of anovulation, implantation failure, hormonal failure, chromosomal abnormalities, or low sperm count in the male partner. Preliminary tests that can be initiated by the nurse practitioner include the following: pelvic examination to ensure that the cervix is open and that there are no uterine abnormalities, basal body temperature charts to plot monthly ovulation and menstrual patterns, and laboratory testing that includes TSH, FSH, LH, and estradiol.

Whenever a patient presents with acute non-traumatic shoulder pain, the clinician should make sure to exclude a: A. Cardiac origin of symptoms B. Gastrointestinal condition C. Cervical spine disorder D. All of the above

D. All of the aboveThe rotator cuff consists of the supraspinatus, infraspinatus, subscapularis, and teres minor. Injury to the rotator cuff is typically due to chronic impingement with degenerative changes over time. The patient typically complains of anterior and lateral shoulder pain that increases with arm elevation and reaching overhead. The pain is usually progressive and may be associated with repetitive activities. Pain at night may cause sleep disturbance. ROM is typically preserved. Apley's (see Fig. 14.2) and Hawkins' (see Fig. 14.3) tests may reproduce the pain, depending on the component of the rotator cuff involved. There may be point or diffuse tenderness to the shoulder area. Crepitus and/or arm weakness suggest an acute tear.

Which of the following serological diagnostic tests is most specific for rheumatoid arthritis? A. C-reactive protein (CRP) B. Rheumatoid factor (RF) C. Anti-nuclear antibodies (ANA) D. Anti-citrullinated protein autoantibodies (ACPA)

D. Anti-citrullinated protein autoantibodies (ACPA) A variety of laboratory tests are used to diagnose RA, including the rheumatoid factor, which is positive in up to 80% of persons with RA but not specific to this disorder. It is often falsely positive in patients with other diseases, including lupus, sarcoidosis, and syphilis. RA is often associated with normocytic, hypochromic anemia, as well as elevations in sedimentation rate and C-reactive protein. Other laboratory tests that may be positive at diagnosis include antinuclear antibody (ANA) and anticitrullinated protein (anti-CP) autoantibodies. The anti-CP autoantibodies are more specific to RA than the rheumatoid factor. Radiological images show loss of joint space and erosion.

An 8-year-old child presents to the emergency room with a severe unilateral, throbbing headache. She is lying on the gurney with her eyes closed while holding her head. She denies ever having this kind of headache in the past. She complains of sensitivity to light and noise. An appropriate history question to ask the parent would be: A. What do you think triggered the headache? B. Have you had a fever? C. Does anyone in your family suffer from migraine headache? D. B and C

D. B and C A migraine is one of the most common types of vascular headache and accounts for a significant percentage of clinic and emergency department visits each year. They occur more frequently in women and the majority of patients report a family history of migraine. Onset of migraine is uncommon after the age of 40 years. They generally do not occur daily and are often associated with the menstrual cycle. Typical migraine pain begins unilaterally but may become generalized and may lateralize to the opposite side and/or radiate to the face or neck. The pain ranges from a dull ache to a throbbing or pulsatile pain. The pain is often severe and/or incapacitating and is often aggravated by movement, light, and noise. Accompanying symptoms may include nausea, vomiting, photophobia, phonophobia, osmophobia, dizziness, chills, and/or ataxia.

A 38-year-old female complains of having trouble with conception. It is important to ask the patient to: A. Schedule a date for pelvic exam and Pap smear B. Chart at least three menstrual cycles C. Take basal body temperature for 3 cycles D. B and C

D. B and C Infertility may be caused by anovulation, decreased function of the corpus luteum, or blocked or scarred fallopian tubes. Any of these can occur despite the fact that the patient may be having menstrual cycles. A history of menstrual irregularities may suggest anovulation or thyroid disease, either of which can cause infertility. A history of STDs could lead to scarring of the tubes. If the patient has not already done so, ask her to chart her menstrual cycles and basal body temperatures for 3 months to determine if and when ovulation is occurring.

A condition that increases risk of breast cancer in a male is: A. Pseudogynecomastia B. BRCA 1 and 2 C. Klinefelter's syndrome D. B and C

D. B and C It is anticipated that over 2,000 new cases of male breast cancer will be diagnosed in 2013, with over 400 related deaths (ACS 2013). Men develop the same types of breast cancer as women. Therefore, it is important to include malignancy in the differential diagnosis when a man complains of breast enlargement. Factors that increase the risk of breast cancer in men include a previous history of breast or testicular disease and Klinefelter's syndrome. A history of gynecomastia is not associated with an increased risk for breast cancer.

Your 77-year-old patient complains of frequent urination, hesitation in getting the stream started, and nocturnal frequency of urination that is bothersome. On DRE, there is an enlarged, firm, non- tender, smooth prostate. The clinician should recognize these as symptoms of: A. Prostatitis B. Prostate cancer C. Urethritis D. Benign prostatic hyperplasia

D. Benign prostatic hyperplasia BPH is an enlargement of the transition zone of the prostate gland which occurs as men age. An enlarged prostate causes symptoms that include urinary urgency, frequency, hesitation in getting the stream started, decreased caliber and force of stream, and nocturnal frequency of urination that is bothersome. This collection of symptoms has also been termed prostatism. A patient with BPH shows symmetric or asymmetric enlargement and a firm, smooth, non-tender gland.

A 23-year-old female presents with episodic bilateral breast tenderness. Upon palpation, there are multiple areas of nodularity with no dominant breast mass and no tenderness, nipple, or skin changes. It is best to recommend: A. Ultrasound of the breasts B. Mammogram of the breasts C. Fine needle aspiration of one nodule within a breast D. Diary of menstrual cycle and breast symptoms

D. Diary of menstrual cycle and breast symptoms Women who experience cyclic mastalgia usually have onset as a teen or young adult. It is important to determine menstrual and reproductive history and to identify all pharmacologic agents taken. A complete breast examination should be performed. The pain associated with hormonal fluctuation most commonly occurs during the second half of the woman's cycle. The variability of the signs and symptoms is identified with a symptom diary and record of the menstrual cycle. The pain is typically poorly localized, bilateral, and nonspecific. It may be accompanied by a sense of breast fullness. The examination may identify the multiple, bilateral nodularities associated with fibroadenomas or fibrocystic changes.

A 16-year-old female complains of sudden left-sided lower quadrant abdominal pain. Last menstrual period was 2 weeks ago. On physical examination, the abdomen is non-tender with no masses and no organomegaly. Bowel sounds are normal in 4 quadrants. Pelvic examination is within normal limits. Abdominal CT scan is negative. Pregnancy test is negative. These signs and symptoms are consistent with: A. Ovarian cancer B. Hydatidiform mole C. Endometriosis D. Follicular cyst

D. Follicular cyst Ovarian cysts are common in the younger population and tend to occur in the latter half of the menstrual cycle. Many spontaneously resolve. Others may need surgical intervention if they become twisted, owing to the risk of gangrene. Right or left lower quadrant pain is usually the presenting complaint. Pelvic examination reveals significant tenderness in the affected adnexal area. Rebound tenderness may be present. Follicular cysts are most common, and the pain typically occurs in the second half of the menstrual cycle. These cysts usually resolve with menses and require no further treatment.

In order to test abstract thought on the mental status exam, the clinician can ask: A. Count by serial 7's backward B. Spell the word "world" backward C. Draw a clock that shows 2:30 D. How is your brother-in-law related to you?

D. How is your brother-in-law related to you? Rationale: Box 15.1 Mental Status Components: - Orientation: The patient should normally be aware of person, date, and place. Ask the patient his or her full name, current date, and place in which the examination is being done. - Memory: Recent and remote memory should normally be intact. Ask what the patient had for lunch yesterday (recent) and where he or she graduated from elementary school (remote). - Fund of knowledge (take into consideration the patient's level of education): Ask about any recent news events or significant upcoming or past holiday. - Attention span: Ability to focus on the interviewer without being easily distracted. Ask the patient to repeat a short list of numbers (e.g., 7-8-9-3-0-2). Inability to repeat six or more numbers indicates attention deficit. - Concentration: Ability to concentrate on a question or task. Ask the patient to remember three unrelated words (red, happy, and five) and then to repeat them in 5 minutes, or ask the patient to count backward from 100 by 7. - Language: Use and understanding of language. Ask the patient to write a full sentence or to spell world backward. Distinguish between dysphonias and dysarthrias, as these indicate mechanical disturbances often due to CN dysfunction. Assess fluency of speech by asking the patient to repeat "no ifs, ands, or buts about it." Dysfluent speech is Broca's aphasia. Speech that is devoid of content indicates Wernicke's aphasia. - Abstract thoughts: Ask the patient to interpret a common proverb (e.g., a stitch in time saves nine), or ask the patient to answer an abstract question (e.g., "Is my sister's brother a man or a woman?").

Your postmenopausal 60-year-old female patient complains of vaginal irritation and pruritus of the external genitalia. She self-medicated with an over-the-counter anti-candida agent. However, the vaginal symptoms persist. The clinician should recognize that: A. Over-the-counter medications do not treat all species of candida B. Postmenopausal women are susceptible to trichomonas infection C. Decreased estrogen makes vaginal mucosa susceptible to bacterial vaginosis D. Irritation due to atrophic vaginal mucosa can mimic symptoms of candida

D. Irritation due to atrophic vaginal mucosa can mimic symptoms of candida Although atrophic vaginitis is not an infection and does not cause a discharge, its signs and symptoms can mimic the C. albicans yeast vaginitis. It occurs in postmenopausal women as a result of a lack of estrogen.

Which of the following variables is not a component of the Gail Model? A. Age at menarche B. Number of breast biopsies C. Age at first live birth D. Number of live births

D. Number of live births The Gail model is a clinical prediction rule used to estimate a patient's risk for breast cancer. The model identifies the relative risk associated with three factors obtained through history: age at menarche, number of previous breast biopsies, and the age at first live birth.

Your 24-year-old patient complains of vaginal discharge. When deciding what diagnostics to perform, what is most important to ascertain? A. Color of the discharge B. Timing of the menstrual cycle C. Consistency of the discharge D. Sexual activity and safe sex practices

D. Sexual activity and safe sex practices If the patient complains about vaginal discharge, it is important to determine if the patient is sexually active, how many partners they have had, and their use of condoms. That will lead you to which diagnostics should be performed. The history should also include amount, color, consistency, odor, itching, burning, inflammation, or lesions, and history of STDs.

Your 55-year-old male patient presents to the emergency department with complaints of sudden development of severe right-sided, colicky lower abdominal pain. He cannot sit still on the examining table. The patient has previously been in good health. On physical examination, there are no signs of peritoneal inflammation. A urine sample reveals hematuria and crystalluria. Which is the next diagnostic test that should be done immediately? A. Ultrasound of the abdomen B. Abdominal x-ray C. Digital rectal examination D. Spiral CT scan

D. Spiral CT scan The initial study can be a KUB or IVP; however, many facilities can perform a stone protocol spiral CT, a much more definitive test for the evaluation of kidney stones. CT can demonstrate filling defects. CT scan is a highly specific and sensitive test for urinary tract calculi. After the initial evaluation with CT scan, pain reliever can be administered. An IVP can be done at another time when the patient can better prepare for the test.

On pelvic examination, you observe a non-tender, ulcerated, red lesion with raised borders on a patient's labia. Her inguinal lymph nodes are enlarged but non-tender. The patient reports that her partner who is HIV+ has a similar lesion on his penis. The clinician should have a high suspicion of: A. Herpes simplex I B. Herpes simplex II C. Condyloma acuminata D. Syphilis

D. Syphilis The incidence of syphilis has risen dramatically in the last 30 years with the increase in HIV and other STDs. In syphilis, the primary chancre lesion appears as a firm, indurated, painless papule that erodes into an ulcer with raised or reddened borders. Chances are usually single lesions and can occur on any mucous membrane or skin area. Nontender lymphadenopathy is present in the regional nodes. Genital lesions are most commonly seen in women on the external genitalia. Symptoms may be mild enough to go unnoticed, especially when they are in areas other than the genitalia, and they heal without treatment in 4 to 8 weeks.

A 23-year-old woman is concerned about a mass she found on palpation. Upon examination of the patient, the mass feels cystic, round, and mobile with discrete borders. The diagnostic study that is recommended is: A. CT scan B. Mammogram C. Biopsy D. Ultrasound

D. Ultrasound The American College of Radiology (2012) recommends that women under 40 years of age should be assessed through ultrasound, and women who are older than 40 years should be assessed with a mammogram. Younger women have denser breasts, and ultrasound is, therefore, often more useful than a mammogram. Ultrasounds are helpful in determining whether or not a mass that feels potentially "cystic" is fluid filled or solid. It is crucial that the woman understand that the imaging procedure is a screening tool and is never diagnostic regarding the existence or absence of a malignancy.

A 55-year-old patient complains of lower back pain due to heavy lifting at work yesterday. He reports weakness of the left leg and paresthesias in the left foot. On physical examination, the patient has diminished ability to dorsiflex the left ankle. Which of the following symptoms should prompt the clinician to make immediate referral to a neurosurgeon? A. Straight leg raising sign B. Lumbar herniated disc on x-ray C. Loss of left sided patellar reflex D. Urinary incontinence

D. Urinary incontinence With low back pain in a patient, it is important to obtain a detailed history of the onset and progression of the pain. A thorough pain history should be completed, noting its quality, location, radiation, and intensity as well as any exacerbating and relieving factors. A thorough review of systems is necessary to identify any associated symptoms that may indicate an urgent problem. These include altered bowel and/or bladder function, fever, weight loss, and/or weakness. The physical examination should begin by noting the patient's posture and apparent level of comfort. The standing patient should be directed through a series of maneuvers to assess the back motion, including flexion, hyperextension, lateral flexion, and rotation, as the smoothness of motion, ROM, and any obvious signs of discomfort are noted. Observe the patient walking on heels and on toes, noting any signs of weakness. Next, with the patient resting supine on the examination table, the straight leg maneuver should be performed. As the patient rests supine with both legs extended, the examiner should passively elevate one leg at a time. A positive test is indicated if the patient experiences discomfort with the initial elevation rather than once the hip has been hyperflexed beyond 50 degrees. If the results indicate nerve impingement or disk injury, further radiographic testing is then indicated.

True or False? Cranial nerves are directly evaluated in the neonate.

False

True or False? Decreased muscle mass increases the risk of damage to connective tissue and bones.

True

True or False? From flexion to extension, a patient should have a 160-degree range of motion in the wrist.

True

True or False? A newborn's urine is usually colorless and odorless.

True

What are the numbers and names of the cranial nerves that function to bring the sense of taste to the brain?

VII: facial IX: glossopharyngeal


Related study sets

CNA Quizes elder abuse-chapter 4

View Set

Ch. 65 Assessment of the Renal/Urinary System

View Set

APBIO: UNIT 2: PRACTICE QUESTIONS!

View Set

Life and Health License - IL (Chapter 3)

View Set

5 functions of the cell membrane

View Set